You are on page 1of 73

www.nucleuseducation.

in

IIT MATHEMATICS
VECTOR & 3D

Corporate Office: NAIVEDHYAM, Plot No. SP-11, Old INOX, Indra Vihar, Kota (Raj.) 324005
Call: 0744-2799900 Mob. 97831-97831, 70732-22177, 0744-2423333
RESULTS OF BEST MENTORSHIP BY THE NUCLEUS TEAM

AIR-1 AIR-3 AIR-6 AIR-8


CHITRAANG MURDIA GOVIND LAHOTI NISHIT AGARWAL AMEY GUPTA
Gen. Category - 2014 Gen. Category - 2014 Gen. Category - 2012 Gen. Category - 2014

JEE MAIN RESULTS 2020 (January Attempt) OF NUCLEUS EDUCATION

100 Percentile 100 Percentile 100 Percentile 100 Percentile 100 Percentile 100 Percentile
(Physics) (Maths & Physics) (Physics) (Maths) (Maths) (Maths)
DAKSH KHANDELWAL VAIBHAV SAHA ANISH MOHAN ARCHIT PATNAIK SWAPNIL YASASVI PARSHANT ARORA
2020 2020 2020 2020 2020 2020

JEE ADVANCED RESULTS OF NUCLEUS EDUCATION

AIR-10 AIR-12 AIR-23 AIR-24 AIR-37 AIR-42 AIR-66 AIR-98


LAKSHAY SHARMA YATEESH AGRAWAL ABHEY GOYAL TUSHAR GAUTAM PIYUSH TIBAREWAL SATVIK MAYANK DUBEY HRITHIK
Gen. Category - 2017 Gen. Category - 2017 Gen. Category - 2017 Gen. Category - 2017 Gen. Category - 2017 Gen. Category - 2017 Gen. Category - 2017 Gen. Category - 2017

AIR-20 AIR-27 AIR-32 AIR-61 AIR-67 AIR-78 AIR-61 AIR-91


SHASHANK AGRAWAL RAAGHAV RAAJ SHREYA PATHAK SIDDHANT CHOUDAHRY ANISWAR S KRISHNAN AAYUSH KADAM SARTHAK BEHERA ANDREWS G. VARGHESE
Gen. Category - 2018 Gen. Category - 2018 Gen. Category - 2018 Gen. Category - 2018 Gen. Category - 2018 Gen. Category - 2018 Gen. Category - 2018 Gen. Category - 2018
DLP *SDCCP *SDCCP

AIR-2 AIR-19 AIR-33 AIR-48 AIR-51 AIR-53 AIR-86


HIMANSHU GAURAV SINGH VIBHAV AGGARWAL S. PRAJEETH SOHAM MISTRI SAYANTAN DHAR GAURAV KRISHAN GUPTA SATVIK JAIN
Gen. Category - 2019 Gen. Category - 2019 Gen. Category - 2019 Gen. Category - 2019 Gen. Category - 2019 Gen. Category - 2019 Gen. Category - 2019
*SDCCP *SDCCP DLP DLP DLP
VECTOR
EXERCISE # 1
STRAIGHT OBJECTIVE TYPE
1. A (1, –1, –3), B (2, 1, –2) & C (–5, 2, –6) are the position vectors of the vertices of a triangle
ABC. The length of the bisector of its internal angle at A is :
(A) 10 /4 (B) 3 10 /4 (C) 10 (D) none
 
2. Let p is the p.v. of the orthocentre & g is the p.v. of the centroid of the triangle ABC where
 
circumcentre is the origin. If p = K g , then K =
(A) 3 (B) 2 (C) 1/3 (D) 2/3

3. A vector a has components 2p & 1 with respect to a rectangular cartesian system. The system is
rotated through a certain angle about the origin in the counterclockwise sense. If with respect to

the new system, a has components p + 1 & 1 then,
(A) p = 0 (B) p = 1 or p = –1/3 (C) p = –1 or p = 1/3 (D) p = 1 or p = –1
 
4. The number of vectors of unit length perpendicular to vectors a = (1, 1, 0) & b (0, 1, 1) is:
(A) 1 (B) 2 (C) 3 (D) 

5. Four points A(+1, –1, 1) ; B(1, 3, 1) ; C(4, 3, 1) and D(4, – 1, 1) taken in order are the vertices of
(A) a parallelogram which is neither a rectangle nor a rhombus
(B) rhombus
(C) an isosceles trapezium
(D) a cyclic quadrilateral.

6. Let , &  be distinct real numbers. The points whose position vector's are  î + ˆj +  k̂ 
 î +  ˆj +  k̂ and  î +  ˆj +  k̂
(A) are collinear (B) form an equilateral triangle
(C) form a scalene triangle (D) form a right angled triangle
  
7. If the vectors a = 3 î + ˆj – 2 k̂ , b = – î + 3 ˆj + 4 k̂ and c = 4 î – 2ˆj – 6kˆ constitute the sides

of a ABC, then the length of the median bisecting the vector c is
(A) 2 (B) 14 (C) 74 (D) 6

8. Let A(0, –1, 1), B(0, 0, 1), C(1, 0, 1) are the vertices of a ABC. If R and r denotes the
r
circumradius and inradius of ABC, then has value equal to
R
3 3  
(A) tan (B) cot (C) tan (D) cot
8 8 12 12

Corporate Office: NAIVEDHYAM, Plot No. SP-11, Old INOX, Indra Vihar, Kota (Raj.) 324005 1
Call: 0744-2799900 Online Partner UNACADEMY
    
9. a, b, c are three non-zero vectors, no two of which are collinear and the vector a  b is
      
collinear with c, b  c is collinear with a , then a  b  c is equal to -
  
(A) a (B) b (C) c (D) none of these

10. If the three points with position vectors (1, a, b) ; (a, 2, b) and (a, b, 3) are collinear in space,
then the value of a + b is
(A) 3 (B) 4 (C) 5 (D) none

11. Consider the following 3 lines in space


L1 : r = 3iˆ  ˆj  2kˆ   (2 ˆi  4 ˆj k)
ˆ

L2 : r = ˆi  ˆj  3k
ˆ   (4iˆ 2 ˆj 4k)
ˆ

L3 : r = 3iˆ  2jˆ  2k
ˆ  t (2iˆ ˆj 2k)
ˆ
Then which one of the following pair(s) are in the same plane.
(A) only L1L2 (B) only L2L3 (C) only L3L1 (D) L1L2 and L2L3

12. The acute angle between the medians drawn from the acute angles of an isosceles right angled
triangle is:
(A) cos–1(2/3) (B) cos–1(3/4) (C) cos–1 (4/5) (D) none

13. The vectors 3iˆ  2ˆj  kˆ , ˆi  3jˆ  5kˆ and 2iˆ  ˆj  4kˆ form the sides of a triangle. Then triangle is
(A) an acute angled triangle (B) an obtuse angled triangle
(C) an equilateral triangle (D) a right angled triangle

14. If the vectors 3p  q;5p  3q and 2p  q; 4p  2q are pairs of mutually perpendicular vectors
then sin ( p q) is
(A) 55 / 4 (B) 55 / 8 (C) 3/16 (D) 247 /16

15.   
Consider the points A, B and C with position vectors 2iˆ  3jˆ  5kˆ , ˆi  2jˆ  3kˆ and 7iˆ  kˆ
respectively.
   
Statement-1: The vector sum, AB + BC + CA = 0
Because
Statement-2: A, B and C form the vertices of a triangle.
(A) Statement-1 is true, statement-2 is true and statement-2 is correct explanation for statement-1.
(B) Statement-1 is true, statement-2 is true and statement-2 is NOT the correct explanation for
statement-1.
(C) Statement-1 is true, statement-2 is false.
(D) Statement-1 is false, statement-2 is true.

Corporate Office: NAIVEDHYAM, Plot No. SP-11, Old INOX, Indra Vihar, Kota (Raj.) 324005 2
Call: 0744-2799900 Online Partner UNACADEMY
16. The set of values of c for which the angle between the vectors cx ˆi  6jˆ  3kˆ and xiˆ  2jˆ  2cxkˆ is
acute for every x  R is
(A) (0, 4/3) (B) [0, 4/3] (C) (11/9, 4/3) (D) [0, 4/3)

 
17. Let u  ˆi  ˆj, v  ˆi  ˆj and w
 ˆ
 i  2jˆ  3kˆ . If n̂ is a unit vector such that u.nˆ = 0 and v.nˆ = 0,

then | w.nˆ | is equal to
(A) 1 (B) 2 (C) 3 (D) 0

18. If the vector 6iˆ  3jˆ  6kˆ is decomposed into vectors parallel and perpendicular to the vector
ˆi  ˆj  kˆ then the vectors are :

 
(A) – ˆi  ˆj  kˆ and 7iˆ  2ˆj  5kˆ  
(B) –2 ˆi  ˆj  kˆ and 8iˆ  ˆj  4kˆ

 
(C) + 2 ˆi  ˆj  kˆ and 4iˆ  5jˆ  8kˆ (D) none

       
19. Let r  a    and r  b  m be two lines in space where a  5iˆ  ˆj  2k, ˆ b  ˆi  7ˆj  8kˆ ,
 
  4iˆ  ˆj  kˆ and m  2iˆ  5jˆ  7kˆ then the p.v. of a point which lies on both of these lines, is
(A) ˆi  2ˆj  kˆ (B) 2iˆ  ˆj  kˆ
(C) ˆi  ˆj  2kˆ (D) non existent as the lines are skew

20. Let A(1, 2, 3), B(0, 0, 1), C(–1, 1, 1) are the vertices of a ABC.
(i) The equation of internal angle bisector through A to side BC is
(A) r = ˆi  2jˆ  3kˆ   (3iˆ 2 ˆj 3k)
ˆ (B) r = (iˆ 2 ˆj 3k)
ˆ  (3iˆ 4jˆ  3k)
ˆ
 
(C) r = ˆi  2jˆ  3kˆ   (3iˆ 3 ˆj 2k)
ˆ (D) r = ˆi  2jˆ  3kˆ   (3iˆ 3 ˆj 4k)
ˆ

(ii) The equation of median through C to side AB is


(A) r = – ˆi  ˆj  kˆ  p (3iˆ 2 k)
ˆ (B) r = – ˆi  ˆj  kˆ  p (3iˆ 2 k)
ˆ
 
(C) r = – ˆi  ˆj  kˆ  p (3iˆ  2k)
ˆ (D) r = – ˆi  ˆj  kˆ  p (3iˆ 2ˆj)

(iii) The area (ABC) is equal to


9 17 17 7
(A) (B) (C) (D)
2 2 2 2
      
21. If a  b  c  0, | a | = 3, | b | = 5, | c | = 7 , then the angle between a & b is :
(A) /6 (B) 2 /3 (C) 5 /3 (D)  /3

22.  
A line passes through the point A ˆi  2jˆ  3kˆ and is parallel to the vector V ˆi  ˆj  kˆ . The  
shortest distance from the origin, of the line is -
(A) 2 (B) 4 (C) 5 (D) 6

Corporate Office: NAIVEDHYAM, Plot No. SP-11, Old INOX, Indra Vihar, Kota (Raj.) 324005 3
Call: 0744-2799900 Online Partner UNACADEMY
       
23. Let a, b, c be vectors of length 3, 4, 5 respectively. Let a be perpendicular to b  c, b to c  a
     
and c to a  b . Then a  b  c is :

(A) 2 5 (B) 2 2 (C) 10 5 (D) 5 2


24. The set of values of x for which the angle between the vectors a = xiˆ  3jˆ  kˆ and
 
b = 2x ˆi  xjˆ  kˆ acute and the angle between the vector b and the axis of ordinates is obtuse, is
(A) 1 < x < 2 (B) x > 2 (C) x < 1 (D) x < 0

  15
25. If a vector a of magnitude 50 is collinear with vector b = 6iˆ  8jˆ – k̂ and makes an acute
2
angle with positive z-axis then :
     
(A) a  4b (B) a  4b (C) b  4a (D) none

   
26. A, B, C & D are four points in a plane with pv's a, b, c & d respectively such that
       
    
a  d · b  c = b  d ·  c  a  = 0. Then for the triangle ABC, D is its
(A) incentre (B) circumcentre (C) orthocentre (D) centroid

  
27. a and b are unit vectors inclined to each other at an angle ,  (0, ) and | a  b | <1.Then 
  2   2      3 
(A)  ,  (B)  ,   (C)  0,  (D)  , 
3 3   3   3 4 4 

28. Image of the point P with position vector 7 ˆi  ˆj  2kˆ in the line whose vector equation is,

 
r  9iˆ  5jˆ  5kˆ   ˆi  3jˆ  5kˆ has the position vector

(A) (– 9, 5, 2) (B) (9, 5, – 2) (C) (9, – 5, – 2) (D) none

29. Let a, ˆ cˆ are three unit vectors such that aˆ  bˆ  cˆ is also a unit vector. If pairwise angles
ˆ b,

between aˆ  bˆ  cˆ are 1, 2 and 3 respectively then cos 1 + cos 2 + cos 3 equals
(A) 3 (B) – 3 (C) 1 (D) – 1

8
30. A tangent is drawn to the curve y = at a point A (x1 , y1) , where x1 = 2. The tangent cuts the
x2
 
x-axis at point B. Then the scalar product of the vectors AB and OB is
(A) 3 (B) – 3 (C) 6 (D) – 6

Corporate Office: NAIVEDHYAM, Plot No. SP-11, Old INOX, Indra Vihar, Kota (Raj.) 324005 4
Call: 0744-2799900 Online Partner UNACADEMY
     
   
 
31. Cosine of an angle between the vectors a  b and a  b if | a | = 2, | b | = 1 and a ^ b = 60°
is
(A) 3/ 7 (B) 9/ 21 (C) 3/ 7 (D) none

32. An arc AC of a circle subtends a right angle at the centre O. The point B divides the arc in the
      
ratio 1 : 2. If OA = a and OB = b , then the vector OC in terms of a & b , is
       
(A) 3 a  2b (B) – 3 a  2b (C) 2a  3 b (D) 2a  3 b

 
33.
  
Given three vectors a, b & c each two of which are non collinear. Further if a  b is collinear  
     
     
with c, b  c is collinear with a & | a | = | b | = | c | = 2 . Then the value of a·b  b·c  c·a :
(A) is 3 (B) is – 3 (C) is 0 (D) cannot be evaluated

34. The vector equations of two lines L1 and L2 are respectively




r = 17i – 9ˆj + 9kˆ +  3iˆ  ˆj  5k


ˆ and r = 15iˆ  8jˆ  kˆ   4iˆ  3jˆ  
I L1 and L2 are skew lines
II (11, –11, –1) is the point of intersection of L1 and L2
III (–11, 11, 1) is the point of intersection of L1 and L2
IV  
cos–1 3 / 35 is the acute angle between L1 and L2
then , which of the following is true?
(A) II and IV (B) I and IV (C) IV only (D) III and IV

     
35. For two particular vectors A and B it is known that A  B = B  A .What must be true about the
two vectors?
(A) At least one of the two vectors must be the zero vector.
   
(B) A  B = B  A is true for any two vectors.
(C) One of the two vectors is a scalar multiple of the other vector.
(D) The two vectors must be perpendicular to each other.

  
36. For some non zero vector V , if the sum of V and the vector obtained from V by rotating it by

an angle 2 equals to the vector obtained from V by rotating it by  then the value of , is
  2 2
(A) 2n ± (B) n ± (C) 2n ± (D) n ±
3 3 3 3
where n is an integer.

Corporate Office: NAIVEDHYAM, Plot No. SP-11, Old INOX, Indra Vihar, Kota (Raj.) 324005 5
Call: 0744-2799900 Online Partner UNACADEMY
       
37. Let u, v, w be such that | u | = 1, | v | = 2, | w | = 3. If the projection of v along u is equal to
      
that of w along u and vectors v , w are perpendicular to each other then | u  v w | equals
(A) 2 (B) 7 (C) 14 (D) 14

 
38. If a and b are non zero, non collinear, and the linear combination
   
(2x – y) a  4b = 5 a + (x – 2y) b holds for real x and y then x + y has the value equal to
(A) – 3 (B) 1 (C) 17 (D) 3

39. Given an equilateral triangle ABC with side length equal to 'a'. Let M and N be two points

   AB  
respectively on the side AB and AC such that AN = KAC and AM = . If BN and CM
3
are orthogonal then the value of K is equal to
1 1 1 1
(A) (B) (C) (D)
5 4 3 2

      
40. If p & s are not perpendicular to each other and rxp = qxp & r.s = 0, then r =
 
   q.p     q.s    
(A) p.s (B) q      p (C) q      p (D) q  p for all scalars 
 p.s   p.s 
     
41. If u and v are two vectors such that | u | = 3; | v | = 2 and | u  v | = 6 then the correct statement
is
         
(A) u ^ v  (0, 90°) (B) u ^ v (90°, 180°) (C) u ^ v = 90° (D) (u × v) × u = 6 v

  


42. Given a parallelogram OACB. The lengths of the vectors OA, OB & AB are a, b & c
 
respectively. The scalar product of the vectors OC & OB is :
a 2  3b2  c2 3a 2  b 2  c2 3a 2  b 2  c2 a 2  3b 2  c2
(A) (B) (C) (D)
2 2 2 2

   
2  
a  3b x 3a  b 
2
43. Vectors a & b make an angle  = . If | a | = 1, | b | = 2 then =
3
(A) 225 (B) 250 (C) 275 (D) 300

 
44. If the vector product of a constant vector OA with a variable vector OB in a fixed plane OAB
be a constant vector, then locus of B is :
 
(A) a straight line perpendicular to OA (B) a circle with centre O radius equal to | OA |

(C) a straight line parallel to OA (D) none of these

Corporate Office: NAIVEDHYAM, Plot No. SP-11, Old INOX, Indra Vihar, Kota (Raj.) 324005 6
Call: 0744-2799900 Online Partner UNACADEMY
       
45. For non-zero vectors a, b, c, a  b.c | a || b || c | holds if and only if ;
   
(A) a.b = 0, b.c = 0 (B) c.a = 0, a.b = 0
    
(C) a.c = 0, b.c = 0 (D) a.b = b.c = c.a = 0

  
46. The vectors a = ˆi  2ˆj  3kˆ ; b = 2iˆ  ˆj  kˆ & c = 3iˆ  ˆj  4kˆ are so placed that the end point of
one vector is the starting point of the next vector. Then the vectors are -
(A) not coplanar
(B) coplanar but cannot form a triangle
(C) coplanar but can form a triangle
(D) coplanar & can form a right angled triangle

47. Given the vectors



u = 2iˆ – ˆj – k̂

v = ˆi  ˆj  2kˆ

w = ˆi  kˆ
  
If the volume of the parallelopiped having – cu , v and c w as concurrent edges, is 8 then 'c' can
be equal to
(A) ± 2 (B) 4 (C) 8 (D) cannot be determined

   
48. ˆ  b) = /2, a·c
Given a = xiˆ  yjˆ  2kˆ , b = ˆi  ˆj  kˆ , c = ˆi  2j;(a = 4 then
    
(A) [a b c]2 = | a | (B) [a b c] = | a | (C) [a b c] = 0 (D) [a b c] = | a |2

  
49. Let a = a1 î + a 2 ˆj + a 3 kˆ ; b  b1ˆi  b2ˆj  b3k;
ˆ c  c ˆi  c ˆj  c kˆ be three non-zero vectors such
1 2 3

     
that c is a unit vector perpendicular to both a & b . If the angle between a & b is , then
6
2
a1 b1 c1
a2 b2 c2 =
a3 b3 c3
(A) 0 (B) 1
1 3
(C) (a12 + a22 + a32) (b12 + b22 + b32) (D) (a12 + a22 + a32) (b12 + b22 + b32) (c12 + c22 + c32)
4 4
  
50. For three vectors u, v, w which of the following expressions is not equal to any of the
remaining three?
           
(A) u· v x w  (B)  v x w ·u (C) v· u x w  (D)  u x v ·w

Corporate Office: NAIVEDHYAM, Plot No. SP-11, Old INOX, Indra Vihar, Kota (Raj.) 324005 7
Call: 0744-2799900 Online Partner UNACADEMY
     
51. Let a = ˆi  ˆj , b  ˆj  kˆ & c = a  b . If the vectors , ˆi  2ˆj  kˆ , 3iˆ  2ˆj  kˆ and c are coplanar

then is

(A) 1 (B) 2 (C) 3 (D) – 3

52. A rigid body rotates with constant angular velocity  about the line whose vector equation is,

 
r   ˆi  2jˆ  2kˆ . The speed of the particle at the instant it passes through the point with p.v.
2iˆ  3jˆ  5kˆ is :

(A)  2 (B) 2 (C)  / 2 (D) none

  
53. Given 3 vectors V1  aiˆ  bjˆ  ck;
ˆ
V2  biˆ  cjˆ  ak;
ˆ V3  ciˆ  ajˆ  bkˆ
  
In which one of the following conditions V1 , V2 and V3 are linearly independent?
(A) a + b + c = 0 and a2 + b2 + c2 ab + bc + ca
(B) a + b + c = 0 and a2 + b2 + c2 = ab + bc + ca
(C) a + b + c  0 and a2 + b2 + c2 = ab + bc + ca
(D) a + b + c  0 and a2 + b2 + c2 ab + bc + ca
     
54. Given unit vectors m, n & p such that angle between m & n = angle between p and
   
 m  n  = then  n p m =
(A) 3/4 (B) 3/4 (C) 1/4 (D) none

  


55. Let AB = 3iˆ  ˆj, AC  2iˆ  3jˆ and DE = 4iˆ  2ˆj . The area of the shaded region in the adjacent
figure, is-

(A) 5 (B) 6 (C) 7 (D) 8


56. The altitude of a parallelopiped whose three coterminous edges are the vectors, A  ˆi  ˆj  k;
ˆ
   
B  2iˆ  4ˆj  kˆ and C  ˆi  ˆj  3kˆ with A and B as the sides of the base of the parallelopiped, is
(A) 2/ 19 (B) 4/ 19 (C) 2 38 /19 (D) none

Corporate Office: NAIVEDHYAM, Plot No. SP-11, Old INOX, Indra Vihar, Kota (Raj.) 324005 8
Call: 0744-2799900 Online Partner UNACADEMY
           
57. Consider ABC with A  (a) ; B  (b) & C  (c) . If b · (a  c) = b · b + a · c ; | b  a | = 3;
   
| c  b | = 4 then the angle between the medians AM and BD is
 1   1 
(A)  – cos–1   (B)  – cos–1  
 5 13   13 5 
 1   1 
(C) cos–1   (D) cos–1  
 5 13   13 5 

58. If A (– 4, 0, 3) ; B (14, 2, –5) then which one of the following points lie on the bisector of the
 
angle between OA and OB ('O' is the origin of reference)
(A) (2, 1, –1) (B) (2, 11, 5) (C) (10, 2, –2) (D) (1, 1, 2)

59. Position vectors of the four angular points of a tetrahedron ABCD are A(3, – 2, 1); B(3, 1, 5);
C(4, 0, 3) and D(1, 0, 0). Acute angle between the plane faces ADC and ABC is
(A) tan–1 (5/ 2) (B) cos–1 (2 /5) (C) cosec–1 (5/ 2) (D) cot–1 (3 /2)
  
60. The volume of the tetrahedron formed by the coterminus edges a, b, c is 3. Then the volume of
     
the parallelopiped formed by the coterminus edges a  b, b  c, c  a is
(A) 6 (B) 18 (C) 36 (D) 9

      
61. If a  ˆi  ˆj  kˆ and b  ˆi  2jˆ  kˆ , then the vector c such that a.c = 2 & a  c  b is -
1

(A) 3iˆ  2jˆ  5kˆ
3

(B)
1 ˆ ˆ ˆ
3
  1

i  2j  5k (C) ˆi  2jˆ  5kˆ
3
 1

(D) 3iˆ  2jˆ  kˆ
3

    
62. a, b and c be three vectors having magnitudes 1, 1 and 2 respectively. If a  (a c) b = 0 ,
 
then the acute angle between a & c is :
(A) /6 (B)  /4 (C)  /3 (D) 5 /12

   
63. If a  ˆi  ˆj  k,
ˆ b  4iˆ  3jˆ  4kˆ and c  ˆi  ˆj  kˆ are linearly dependent vectors & | c | = 3
then
(A)  = 1,  = –1 (B)  = 1,  = ±1 (C)  = –1,  = ±1 (D) = ±1,  = 1

64. A vector of magnitude 5 5 coplanar with vectors ˆi  2ˆjand ˆj  2kˆ and the perpendicular
vector 2iˆ  ˆj  2kˆ is

(A) ± 5 5iˆ  6jˆ  8kˆ  (B) ± 5 5iˆ  6jˆ  8kˆ 

(C) ± 5 5 5iˆ  6jˆ  8kˆ  (D) ±  5iˆ  6jˆ  8kˆ 
  
65. Let  = 2iˆ  3jˆ  kˆ and   ˆi  ˆj . If  is a unit vector, then the maximum value of
   
       is equal to
 
(A) 2 (B) 3 (C) 4 (D) 9

Corporate Office: NAIVEDHYAM, Plot No. SP-11, Old INOX, Indra Vihar, Kota (Raj.) 324005 9
Call: 0744-2799900 Online Partner UNACADEMY
MATRIX MATCH TYPE
66. If A(0, 1, 0), B(0, 0, 0), C(1, 0, 1) are the vertices of a ABC. Match the entries of column-I
with column-II.
Column-I Column-II
2
(A) Orthocentre of ABC. (P)
2
3
(B) Circumcentre of ABC. (Q)
2
3
(C) Area (ABC). (R)
3
3
(D) Distance between orthocentre and centroid. (S)
6
(E) Distance between orthocentre and (T) (0, 0, 0)
circumcentre.
1 1 1
(F) Distance between circumcentre and (U)  , , 
2 2 2
centroid.
1 1 1
(G) Incentre of ABC. (V)  , , 
3 3 3
(H) Centroid of ABC (W)  1 2 1 
 1  2  3 , 1  2  3 , 1  2  3 
 

Corporate Office: NAIVEDHYAM, Plot No. SP-11, Old INOX, Indra Vihar, Kota (Raj.) 324005 10
Call: 0744-2799900 Online Partner UNACADEMY
EXERCISE # 2

1. Given the vector PQ   6iˆ  4jˆ and Q is the point (3, 3), find the point P.

2. Find the unit vector (in xy plane) obtained by rotating j counterclockwise 3/4 radian about the
origin.

3. Show that the vector v = ai + bj is perpendicular to the line ax + by = c.



4. In ABC, a point P is chosen on side AB so that AP : PB = 1 : 4 and a point Q is chosen on
   MC
the side BC so that CQ : QB = 1 : 3. Segment CP and AQ intersect at M. If the ratio is
PC
a
expressed as a rational numbers in the lowest term as , find (a + b).
b
   
5. Let O be an interior point of ABC such that 2OA + 5OB + 10OC = 0 . If the ratio of the
area of ABC to the area of AOC is t, where ‘O’ is the origin. Find [t].
(where [ ] denotes greatest integer function)

6. If the distance from the point P(1, 1, 1) to the line passing through the points Q(0, 6, 8) and
R(–1, 4, 7) is expressed in the form p / q where p and q are coprime, then the value of
(p q)(p q  1)
.
2
7. Let S(t) be the area of the OAB with O(0, 0, 0), A (2, 2, 1) and B(t, 1, t + 1).
 e3  a 
e

The value of the definite integral  (S(t))2 ntdt , is equal to   where a, b  N, find (a + b).
1  b 

8. Given f2(x) + g2(x) + h2(x)  9 and U(x) = 3f(x) + 4g(x) + 10h(x), where f(x), g(x) and h(x) are
continuous  x  R. If maximum value of U(x) is N , then find N.

    
9. If a and b are non collinear vectors such that p = (x  4 y) a + (2x + y + 1) b and
    
q = (y – 2x + 2) a + (2 x  3 y 1) b , find x and y such that 3 p = 2 q .

       
10. (a) Show that the points a – 2 b + 3 c ; 2 a + 3 b – 4 c and –7 b + 10 c are collinear.

(b) Prove that the points A(1, 2, 3), B(3, 4, 7), C(–3, –2, –5) are collinear and find the ratio in
which B divides AC.

Corporate Office: NAIVEDHYAM, Plot No. SP-11, Old INOX, Indra Vihar, Kota (Raj.) 324005 11
Call: 0744-2799900 Online Partner UNACADEMY
11. Find out whether the following pairs of lines are parallel, non-parallel & intersecting, or
nonparallel and non-intersecting.
 ˆ ˆ ˆ  ˆ ˆ ˆ
r1  i  j  2k  (3iˆ 2jˆ  4k) ˆ r  i  j  3k  (iˆ ˆj  k) ˆ
(a)  (b)  1
r2  2iˆ  ˆj  3kˆ  (6 ˆi  4jˆ  8k)
ˆ r2  2iˆ  4jˆ  6kˆ  (2 ˆi  ˆj  3k)
ˆ
 ˆ ˆ
r  i  k  (iˆ 3jˆ  k) ˆ
(c)  1
r2  2iˆ  3jˆ  (4 ˆi  ˆj  k)
ˆ

   
12. If and s are non zero constant vectors and the scalar b is chosen such that | r  bs | is
r
   
minimum, then show that the value of | bs |2  | r  bs |2 is equal to | r |2 .

13. In a unit cube. Find


(a) The angle between the diagonal of the cube and a diagonal of a face skew to it.
(b) The angle between the diagonals of two faces of the cube through the same vertex.
(c) The angle between a diagonal of a cube and a diagonal of a face intersecting it.

Instruction for question nos. 14 to 16 :


  
Suppose the three vectors a, b, c on a plane satisfy the condition that
       
| a || b || c | = | a  b | = 1; c is perpendicular to a and b·c > 0, then

  
14. Find the angle formed by 2a  b and b.
  
15. If the vector c is expressed as a linear combination a  b then find the ordered pair (,).

    
16. For real numbers x,y the vector p  xa  yc satisfies the condition 0  p·a  1 and 0  p·b  1 .

Find the maximum value of p·c .

17. (a) Find the minimum area of the triangle whose vertices are A(–1,1,2); B(1,2,3) and C(t,1,1)
where t is a real number.
      
(b) Let OA = a ; OB = 100 a + 2 b and OC = b where O, A and C are non collinear points.
 
Let P denotes the area of the parallelogram with OA and OC as adjacent sides and Q
denotes the area of the quadrilateral OABC. If Q = P. Find the value of .

    1 
18. Given that a and b are two unit vectors such that angle between a and b is cos–1   . If c be a
4
         
vector in the plane of a and b , such that | c | = 4, c  b = 2a  b and c  a  b then, find
(a) the value of , (b) the sum of values of  and (c) the product of all possible values of .

Corporate Office: NAIVEDHYAM, Plot No. SP-11, Old INOX, Indra Vihar, Kota (Raj.) 324005 12
Call: 0744-2799900 Online Partner UNACADEMY
  
19. Let A  ˆi  2jˆ  3k,
ˆ B  2iˆ  ˆj  kˆ , C  ˆj  kˆ .
      
If the vector B  C can be expressed as a linear combination B  C = x A + y B + zC where x,
y, z are scalars, then find the value of (100x + 10y + 8z).

         
20. The base vectors a1 , a 2 , a 3 are given in terms of base vectors b1 , b2 , b3 as a1 = 2b1  3b 2  b3 ;
            
a 2  b1  2b2  2b3 and a 3 = 2b1  b2  2b3 . If F = 3b1  b2  2b3 , then express F in terms of
  
a1 , a 2 and a 3 .


21. The vector OP = ˆi  2jˆ  2kˆ turns through a right angle, passing through the positive x-axis on
the way. Find the vector in its new position.

22.     
The pv's of the four angular points of a tetrahedron are A ˆj  2kˆ ; B 3iˆ  kˆ ; C 4iˆ  3jˆ  6kˆ & 
 
D 2iˆ  3jˆ  2kˆ . Find :
(i) the perpendicular distance from A to the line BC.
(ii) the volume of the tetrahedron ABCD.
(iii) the perpendicular distance from D to the plane ABC.
(iv) the shortest distance between the lines AB & CD.
  
23.  
Let a 3 dimensional vector V satisfies the condition 2V  V  ˆi  2jˆ = 2iˆ  kˆ .

If 3| V | = m , where m  N, then find m.
 
24. If x, y are two non-zero and non-collinear vectors satisfying
  
[(a – 2)2 + (b – 3) + c] x + [(a – 2)2 + (b – 3) + c] y + [(a – 2)2 + (b – 3) + c] (x  y) = 0
where , ,  are three distinct real numbers, then find the value of (a2 + b2 + c2).

25. Solve the simultaneous vector equations for the vectors x and y .
        
x  c  y  a and y  c  x  b where c is a non zero vector.

  
26. Vector V is perpendicular to the plane of vectors a  2iˆ  3jˆ  kˆ and b  ˆi  2ˆj  3kˆ and

  
satisfies the condition V. ˆi  2jˆ  7kˆ = 10. Find | V |2 .

  2  
27. Let two non-collinear vectors a and b inclined at an angle be such that | a | = 3 and | b | = 4.
3

A point P moves so that at any time t the position vector OP (where O is the origin) is given as
  
OP =  et  e t  a   et  e t  b . If the least distance of P from origin is 2 p  q where
p, q  N then find the value of (p + q).

Corporate Office: NAIVEDHYAM, Plot No. SP-11, Old INOX, Indra Vihar, Kota (Raj.) 324005 13
Call: 0744-2799900 Online Partner UNACADEMY
EXERCISE # 3 (JM)
 
1. ABC is a triangle, right angled at A. The resultant of the forces acting along AB , AC with
1 1 
magnitudes and respectively is the force along AD , where D is the foot of the
AB AC
perpendicular from A onto BC. the magnitude of the resultant is- [AIEEE-2006]

(1)
 AB AC  (2)
1

1
(3)
1
(4)
AB2  AC2
AB  AC  AB  AC 
2 2
AB AC AD

2. If û and v̂ are unit vectors and  is the acute angle between them, then 2 û × 3 v̂ is a unit
vector for- [AIEEE-2007]
(1) Exactly two values of  (2) More than two values of 
(3) No value of  (4) Exactly one value of 

   
3. Let a = î + ˆj + k̂ , b = î – ˆj + 2 k̂ and c = x î + (x – 2) ˆj – k̂ . If the vector c lies in the plane
 
of a and b , then x equals - [AIEEE-2007]
(1) 0 (2) 1 (3) –4 (4) –2

  
4. The vector a =  î + 2 ˆj +  k̂ , lies in the plane the vectors b = î + ˆj and c = ˆj + k̂ and bisect
 
the angle between b and c . Then which one of the following gives possible values of and ?
[AIEEE-2008]
(1) = 2,  = 2 (2)  = 1,  = 2 (3)  = 2,  = –1 (4)  = 1,  = 1

  
5. If u, v, w are non-coplanar vectors and p, q are real numbers, then the equality [AIEEE-2009]
       
3u pv pw  – pv w qu  –  2w qv qu  = 0 holds for :-
(1) More than two but not all values of (p,q)
(2) All values of (p, q)
(3) Exactly one value of (p, q)
(4) Exactly two values of (p, q)

       
6. ˆ Then the vector b satisfying a  b  c = 0 and a·b = 3 is :
Let a  ˆj  kˆ and c  ˆi  ˆj  k.
[AIEEE-2010]
(1) ˆi  ˆj  2kˆ (2) 2iˆ  ˆj  2kˆ (3) ˆi  ˆj  2kˆ (4) ˆi  ˆj  2kˆ

Corporate Office: NAIVEDHYAM, Plot No. SP-11, Old INOX, Indra Vihar, Kota (Raj.) 324005 14
Call: 0744-2799900 Online Partner UNACADEMY
       
7. The vectors a and b are not perpendicular and c and d are two vectors satisfying : b  c  b  d
 
and a.d = 0 then the vector d is equal to :- [AIEEE-2011]
     
  b·c     a·c     b·c     a·c  
(1) b      c (2) c      b (3) b      c (4) c      b
 a·b   a·b   a·b   a·b 

 1      
8.

If a =
1
10
 
3iˆ  kˆ and b =
7
   
2iˆ  3jˆ  6kˆ , then the value of 2a  b · a  b  a  2b  is :
    
[AIEEE-2011]
(1) 5 (2) 3 (3) – 5 (4) – 3

    
9. Let a, b, c be three non-zero vectors which are pairwise non-collinear. If a  3b is collinear with
      
c and b  2c is collinear with a , then a  3b  6c is : [AIEEE-2011]
    
(1) a  c (2) a (3) c (4) 0

 
10. Let â and b̂ be two unit vectors. If the vectors c  aˆ  2bˆ and d  5aˆ  4bˆ are perpendicular to
each other, then the angle between â and b̂ is : [AIEEE-2012]
   
(1) (2) (3) (4)
4 6 2 3
   
11. Let ABCD be a parallelogram such that AB = q, AD  p and BAD be an acute angle. If r is
the vector that coincides with the altitude directed from the vertex B to the side AD, then r is
given by : [AIEEE-2012]
 
 3(p·q)   3(p·q) 
(1) r = – 3 q +   p (2) r = 3q –   p
(p·p) (p·p)
 
  p·q     p·q  
(3) r = – q +     p (4) r = q –     p
 p·p   p·p 

 
12. If the vectors AB = 3iˆ  4kˆ and AC = 5iˆ  2jˆ  4kˆ are the sides of a triangle ABC, then the
length of median through A is :
(1) 18 (2) 72 (3) 33 (4) 45

    
13. ˆ b  ˆi  2jˆ  kˆ and c = ˆi  ˆj  2kˆ be three vectors. A vectors of the type b  c
Let a = 2iˆ  ˆj  k,
 2
for some scalar , whose projection on a is of magnitude , is :
3
[JEE-MAINS Online 2013]
(1) 2iˆ  ˆj  5kˆ (2) 2iˆ  ˆj  5kˆ (3) 2iˆ  ˆj  5kˆ (4) 2iˆ  3jˆ  3kˆ

Corporate Office: NAIVEDHYAM, Plot No. SP-11, Old INOX, Indra Vihar, Kota (Raj.) 324005 15
Call: 0744-2799900 Online Partner UNACADEMY
       
14. Let a = 2iˆ  ˆj  2kˆ , b = ˆi  ˆj . If c is a vector such that a  c = | c | , | c a | = 2 2 and the
    

 
angle between a × b and c is 30º, then a  b  c equals : [JEE-MAINS Online 2013]

3 1 3 3
(1) (2) 3 (3) (4)
2 2 2

     2
15. If a  b b  c c  a  =  a b c  then  is equal to : [JEE(Main)-2014]
(1) 2 (2) 3 (3) 0 (4) 1

  
16. Let a, b and c be three non – zero vectors such that no two of them are collinear and
   1     
 
a  b × c = | b | c | a . If  is the angle between vectors b and c , then a value of sin  is :
3
[JEE(Main)-2015]
2 2 3 2 2  2
(1) (2) (3) (4)
3 3 3 3

   3   
17.
  
Let a, b and c be three unit vectors such that a × b  c =  
2
 
b  c . If b is not parallel to
  
c , then the angle between a and b is :- [JEE(Main)-2016]
5 3  2
(1) (2) (3) (4)
6 4 2 3
  
 
 
  
18. Let a = 2iˆ  ˆj  2kˆ and b  ˆi  ˆj . Let c be a vector such that | c  a | = 3, a  b  c  3 and
   
the angle between c and a  b be 30°. Then a.c is equal to : [JEE (Main)-2017]
1 25
(1) (2) (3) 2 (4) 5
8 8

   
19. Let u be a vector coplanar with the vectors a  2iˆ  3jˆ  kˆ and b  ˆj  kˆ , If u is perpendicular
  2
to a and u.b  24 , then u is equal to [JEE (Main)-2018]
(1) 84 (2) 336 (3) 315 (4) 265

20. The magnitude of the projection of the vector 2iˆ  3jˆ  kˆ on the vector perpendicular to the
plane containing the vectors ˆi  ˆj  kˆ and ˆi  2ˆj  3kˆ is ; [JEE (Main)-2019]
3 3
(1) 6 (2) 3 6 (3) (4)
2 2

Corporate Office: NAIVEDHYAM, Plot No. SP-11, Old INOX, Indra Vihar, Kota (Raj.) 324005 16
Call: 0744-2799900 Online Partner UNACADEMY
21 If a point R(4, y, z) lies on the line segment joining the points P(2, –3, 4) and Q(8, 0, 10), then
the distance of R from the origin is : [JEE (Main)-2019]
(1) 6 (2) 53 (3) 2 14 (4) 2 21

   
22. Let a  3iˆ  2jˆ  xkˆ and b  ˆi  ˆj  kˆ , for some real x. Then a  b  r is possible if :
[JEE (Main)-2019]
3 3 3 3 3 3
(1) r  5 (2) r3 (3) 3 r5 (4) 0  r 
2 2 2 2 2 2

       
23. Let   3iˆ  ˆj and b  2iˆ  ˆj  3kˆ . If   1  2 , where 1 is parallel to  and  2 is
  
perpendicular to  , then 1 2 is equal to: [JEE (Main)-2019]
1 ˆ (4) 1 (3iˆ  9jˆ  5k)
(1) 3iˆ  9jˆ  5kˆ (2) 3iˆ  9jˆ  5kˆ (3) (3iˆ  9jˆ  5k) ˆ
2 2

24. If a unit vector a makes angle /3 with î , /4 with ˆj and  (0, ) with k̂ , then a value of 
is : [JEE (Main)-2019]
5 5  
(1) (2) (3) (4)
6 12 4 3

25. The distance of the point having position vector ˆi  2ˆj  6kˆ from the straight line passing
through the point (2, 3, –4) and parallel to the vector, 6iˆ  3jˆ  4kˆ is : [JEE (Main)-2019]
(1) 6 (2) 2 13 (3) 7 (4) 4 3

26 If the volume of parallelopiped formed by the vectors ˆi  ˆj  kˆ , ˆj  kˆ and ˆi  kˆ is
minimum, then  is equal to [JEE (Main)-2019]
1 1
(1) – 3 (2) (3) – (4) 3
3 3

 
27. Let a  3iˆ  2jˆ  2kˆ and b  ˆi  2jˆ – 2kˆ be two vectors. If a vector perpendicular to both the
   
vectors a + b and a – b has the magnitude 12 then one such vector is: [JEE (Main)-2019]
(1) 4(–2 ˆi– 2 ˆj k)
ˆ (2) 4(2 ˆi– 2 ˆj– k)
ˆ (3) 4(2 ˆi  2 ˆj – k)
ˆ (4) 4(2 ˆi  2 ˆj  k)
ˆ

  
28. Let  R and the three vectors a  ˆi  ˆj  3kˆ , b  2iˆ  ˆj  kˆ and c  ˆi  2ˆj  3kˆ . Then the
  
set S = { : a , b , and c are coplanar} [JEE (Main)-2019]
(1) is singleton
(2) contains exactly two numbers only one of which is positive
(3) is empty
(4) contains exactly two positive numbers

Corporate Office: NAIVEDHYAM, Plot No. SP-11, Old INOX, Indra Vihar, Kota (Raj.) 324005 17
Call: 0744-2799900 Online Partner UNACADEMY
        2
29. Let a  ˆi  ˆj, b  ˆi  ˆj  kˆ and c be a vector such that a  c  b  0 and a.c  4 , then c is
equal to : [JEE (Main)-2019]
19 17
(1) (2) 8 (3) (4) 9
2 2
  
30.  b  b i  b j  2k and c  5i  j  2k be three vectors such that the
Let a  i  j  2k, 1 2
      
projection vector of b on a is a . If a + b is perpendicular of c , then b is equal to :
(1) 22 (2) 4 (3) 6 (4) 32
[JEE (Main)-2019]
  
31. Let a  2iˆ  1ˆj  3kˆ , b  4iˆ  (3   2 ) ˆj  6kˆ and c  3iˆ  6jˆ  (3  1) kˆ be three vectors such that
   
b  2a and a is perpendicular to c . Then a possible value of (1, 2, 3) is: [JEE (Main)-2019]
1   1 
(1)  , 4, 2  (2) (1, 5, 1) (3)   , 4,0  (4) (1, 3, 1)
2   2 

       
32. Let   (  2)a  b and   (4  2)a  3b be two given vectors where vectors a and b are
 
non-collinear. The value of  for which vectors  and  are collinear, is: [JEE (Main)-2019]
(1) –3 (2) 4 (3) –4 (4) 3
  
33. Let a  i  2j  4k,
 
 
 b  i  j  4k and c  2i  4j   2  1 k be coplanar vectors. Then the

non-zero vector a  c is : [JEE (Main)-2019]


(1) 10i  5j (2) 14i  5j (3) 10i  5j (4) 14i  5j

34. Let 3iˆ  ˆj,i  3jˆ and ˆi  (1  ) ˆj respectively be the position vectors of the points A, B and C
with respect to the origin O. If the distance of C from the bisector of the acute angle between
3
OA and OB is , then the sum of all possible values of  is: [JEE (Main)-2019]
2
(1) 3 (2) 4 (3) 1 (4) 2

35. The sum of the distinct real values of  , for which the vectors, i + j  k,
 i  j  k,
 i  j  k
are co–planar, is : [JEE (Main)-2019]
(1) 2 (2) 0 (3) –1 (4) 1
    
36. Let a, b and c be three unit vectors, out of which vectors b and c are non-parallel. If  and 
    1
 

are the angles which vector a makes with vectors b and c respectively and a  b  c  b , then |
2

– | is equal to: [JEE (Main)-2019]
(1) 30º (2) 45º (3) 90º (4) 60º

Corporate Office: NAIVEDHYAM, Plot No. SP-11, Old INOX, Indra Vihar, Kota (Raj.) 324005 18
Call: 0744-2799900 Online Partner UNACADEMY
  
37. A vector a  ˆi  2jˆ  kˆ (,   R) lies in the plane of the vectors, b  ˆi  ˆj and c  ˆi  ˆj  4kˆ .
  
If a bisects the angle between b and c , then : [JEE (Main)-2020]
 ˆ  ˆ  ˆ  ˆ
(1) a  k  4  0 (2) a  i  1  0 (3) a  k  2  0 (4) a  i  3  0

  
38. Let a, b and c be three unit vector such
   
that a  b  c  0 if
     
 = a ·b  b·c  c.a and
      
d  a  b  b  c  c  a then

 
the ordered pair, , d is equal to : [JEE (Main)-2020]
 3    3    3    3  
(1)   ,3a  b  (2)  ,3b  c  (3)   ,3c  b  (4)  ,3a  c 
 2  2   2  2 

39. If the foot of the perpendicular drawn from the point (1, 0, 3) on a line passing through (, 7, 1)
 5 7 17 
is  , ,  , then  is equal to ............ [JEE (Main)-2020]
3 3 3 

40. Let the volume of a paralleleopiped whose coterminous edges are given by u  ˆi  ˆj  kˆ
   
v  ˆi  ˆj  3kˆ and w  2iˆ  ˆj  kˆ be 1 cu. unit. If  be the angle between the edge u and w ,
then cos can be : [JEE (Main)-2020]
7 5 7 5
(1) (2) (3) (4)
6 6 3 3 6 3 7

      
41. Let a  ˆi  2jˆ  kˆ and b  ˆi  ˆj  kˆ be two vectors. If c is a vector such that b  c = b  a and
 
c.a  0 then c.b is equal to [JEE (Main)-2020]
1 3 1
(1) (2) –1 (3)  (4) 
2 2 2

42. If the vectors,



p  (a  1)iˆ  ajˆ  akˆ ,

q  aiˆ  (a  1) ˆj akˆ and
    
r  aiˆ  ajˆ  (a  1)kˆ (a  R) are coplanar and 3(p  q) 2   | r  q |2  0 then value
of  is ............ [JEE (Main)-2020]

       
43. Let a, b and c be three vectors such that | a | 3 , | b | 5 , b  c  10 and the angle between b
       
and c is . If a is perpendicular to the vector b  c , then a  (b  c) is equal to_______.
3
[JEE (Main)-2020]
Corporate Office: NAIVEDHYAM, Plot No. SP-11, Old INOX, Indra Vihar, Kota (Raj.) 324005 19
Call: 0744-2799900 Online Partner UNACADEMY
    2  2  2  2
44. Let a, b and c be three unit vectors such that a  b  a  c  8 . Then a  2b  a  2c is
equal to______. [JEE (Main)-2020]
 
ˆ and r  (2iˆ  ˆj)  m(iˆ  ˆj  k)
ˆ
45. The lines r  (iˆ  ˆj)  l (2iˆ  k) [JEE (Main)-2020]
(1) intersect when l = 1 and m = 2 (2) do not intersect for any values of l and m
1
(3) intersect for all values of l and m (4) intersect when l = 2 and m =
2

 2   4 
46. Let a, b, c  R be such that a2 + b2 + c2 = 1. If a cos = b cos     = cos     , where
 3   3 

 , then the angle between the vectors aiˆ  bjˆ  ckˆ and biˆ  cjˆ  akˆ is : [JEE (Main)-2020]
9
  2
(1) (2) 0 (3) (4)
2 9 3
    
47. Let x0 be the point of local maxima of ƒ(x) = a  (b c) , where a  xiˆ  2jˆ  3kˆ , b  2iˆ  xjˆ  kˆ
      
and c  7iˆ  2jˆ  xkˆ . Then the value of a  b  b  c  c  a at x = x0 is : [JEE (Main)-2020]
(1) –30 (2) –22 (3) 14 (4) –4


 

 
 
 
2 2 2
48. If a  2iˆ  ˆj  2kˆ , then the value ˆi  a  ˆi  ˆj  a  ˆj  kˆ  a  kˆ of is equal to _______.
[JEE (Main)-2020]

49. If the volume of a parallopiped, whose coterminos edges are given by the vectors a  ˆi  ˆj  nkˆ ,
 
b  2iˆ  4jˆ  nkˆ and c  ˆi  njˆ  3kˆ (n  0), is 158 cu. units, Then : [JEE (Main)-2020]
   
(1) a  c  17 (2) b  c  10 (3) n = 9 (4) n = 7

       
50. Let the vectors a, b, c be such that a  2 , b  4 and c  4 . If the projection of b on a is
      
equal to the projection of c on a and b is perpendicular to c , then the value of a  b  c is
______. [JEE (Main)-2020]
     
51. If a and b are unit vectors, then the greatest value of 3 | a  b |  | a  b | is-
[JEE (Main)-2020]
       
52. If x and y be two non-zero vectors such that x  y  x and 2x  y is perpendicular to y ,
then the value of  is _____. [JEE (Main)-2020]

Corporate Office: NAIVEDHYAM, Plot No. SP-11, Old INOX, Indra Vihar, Kota (Raj.) 324005 20
Call: 0744-2799900 Online Partner UNACADEMY
EXERCISE # 4 (JA)
1. Incident ray is along the unit vector v̂ and the reflected ray is along the unit vector .The normal
is along unit vector â outwards. Express ŵ in terms of â and v̂ . [JEE 05 (Mains)4]

   ˆ ˆ ˆ  
2. (a) Let a  ˆi  2jˆ  k, ˆ b  ˆi  ˆj  kˆ and c  i  j  k. A vector in the plane of a and b whose
 1
projection on c has the magnitude equal to is -
3
(A) 4 ˆi  ˆj  4kˆ (B) 3iˆ  ˆj  3kˆ (C) 2iˆ  ˆj  2kˆ (D) 4 ˆi  ˆj  4kˆ

(b) Let A be vector parallel to line of intersection of planes P1 and P2 through origin. P1 is
parallel to the vectors 2 ˆj  3kˆ and 4 ˆj  3kˆ and P2 is parallel to ˆj  kˆ and 3iˆ  3jˆ , then the

angle between vector A and 2iˆ  ˆj  2kˆ is – [JEE 2006, 3+5]
   
(A) (B) (C) (D)
2 4 6 3

3. (a) The number of distinct real values of , for which the vectors 2ˆi  ˆj  k,
ˆ ˆi   2ˆj  kˆ and
ˆi  ˆj   2 kˆ are coplanar, is -
(A) zero (B) one (C) two (D) three
      
(b) Let a, b, c be unit vectors such that a  b  c  0 . Which one of the following is correct?
             
(A) a  b  b  c  c  a  0 (B) a  b  b  c  c  a  0
            
(C) a  b  b  c  a  c  0 (D) a  b, b  c, c  a are mutually perpendicular
     
(c) Let the vectors PQ, QR, RS,ST, TU and UP represent the sides of a regular hexagon.
   
Statement-1 : PQ  (RS  ST)  0 .
because
     
Statement-2 : PQ  RS  0 and PQ  ST  0 .
(A) Statement-1 is True, Statement-2 is True ; Statement-2 is a correct explanation for
Statement-1.
(B) Statement-1 is True, Statement-2 is True ; Statement-2 is NOT a correct explanation
for Statement-1.
(C) Statement-1 is True, Statement-2 is False.
(D) Statement-1 is False, Statement-2 is True. [JEE 2007, 3+3+3]

Corporate Office: NAIVEDHYAM, Plot No. SP-11, Old INOX, Indra Vihar, Kota (Raj.) 324005 21
Call: 0744-2799900 Online Partner UNACADEMY
4. (a) The edges of a parallelopiped are of unit length and are parallel to non-coplanar unit

ˆ b,
vectors a, ˆ·cˆ = cˆ·aˆ = 1 . Then, the volume of the parallelopiped is :
ˆ cˆ such that â·bˆ = b
2
1 1 3 1
(A) (B) (C) (D)
2 2 2 2 3

(b) Let two non-collinear unit vectors â and b̂ form an acute angle. A point P moves so that at

any time t the position vector OP (where O is the origin) is given by â cost + b̂ sin t. when
 
P is farthest from origin O, let M be the length of OP and û be the unit vector along OP .
Then-
â  bˆ â  bˆ
(A) û = and M = (1 + â.bˆ )1/2 (B) û = and M = (1 + â.bˆ )1/2
â  bˆ â  bˆ

â  bˆ â  bˆ
(C) û  and M = (1 + 2 â.bˆ )1/2 (D) û = and M = (1 + 2 â.bˆ )1/2
â  bˆ â  bˆ

[JEE 2008, 3+3]


    
5.
  
  
 1
(a) If a, b, c and d are unit vectors such that a  b . c  d = 1 and a.c  , then –
2
     
(A) a, b, c are non - coplanar (B) b, c, d are non - coplanar
     
(C) b, d are non - parallel (D) a, d are parallel and b, c are parallel

(b) Match the statements / expression given in Column I with the value given in Column II.
Column – I Column - II

(A) Root(s) of the equation 2sin2 + sin22 = 2 (P)
6
 6x   3x  
(B) Points of discontinuity of the function f(x) =   cos   (Q)
  4
where [y] denotes the largest integer less than or equal to y

(C) Volume of the parallelepiped with its edges (R)
3

represented by the vectors ˆi  ˆj, ˆi  2ˆj and ˆi  ˆj  kˆ (S)
2
    
(D) Angle between vectors a and b where a, b and c are unit (T) 
   
vectors satisfying a  b  3c  0 [JEE 2009, 3+8]

Corporate Office: NAIVEDHYAM, Plot No. SP-11, Old INOX, Indra Vihar, Kota (Raj.) 324005 22
Call: 0744-2799900 Online Partner UNACADEMY

6. (a) Two adjacent sides of a parallelogram ABCD are given by AB = 2iˆ  10ˆj  11kˆ and

ˆ The side AD is rotated by an acute angle  in the plane of the
AD = – ˆi  2ˆj  2k.
parallelogram so that AD becomes AD'. If AD' makes a right angle with the side AB then
the cosine of the angle  is given by –
8 17 1 4 5
(A) (B) (C) (D)
9 9 9 9

   ˆi  2ˆj  2iˆ  ˆj  3kˆ


(b) If a and b are vectors in space given by a = and b = , then the value of
5 14
   
 2a  b  .  a  b    a  2b  is

[JEE 2010, 5 + 3]

   
7. (a) Let a = ˆi  ˆj  k,
ˆ b  ˆi  ˆj  kˆ and c  ˆi  ˆj  kˆ be three vectors. A vector v in the plane of
   1
a and b , whose projection on c is , is given by
3
(A) ˆi  3jˆ  3kˆ (B) 3iˆ  3jˆ  kˆ (C) 3iˆ  ˆj  3kˆ (D) ˆi  3jˆ  3kˆ

(b) The vector(s) which is/are coplanar with vectors ˆi  ˆj  2kˆ and ˆi  2ˆj  kˆ , and perpendicular
to the vector ˆi  ˆj  kˆ is/are
(A) ˆj  kˆ (B) ˆi  ˆj (C) ˆi  ˆj (D) ˆj  kˆ

 
ˆ bˆ  ˆi  ˆj and c  ˆi  2ˆj  3kˆ be three given vectors. If r is a vector such
(c) Let a = ˆi  k,
     
that r  b  c  b and r.a = 0, then the value of r.b is [JEE 2011, 3+4+4]

           
8. (a) If a, b and c are unit vectors satisfying | a  b |2 + | b  c | 2 + | c  a | 2 = 9, then | 2 a  5 b 5c | is

     
(b) If a and b are vectors such that | a  b | = 29 and a × (2iˆ 3 ˆj  4k)
ˆ = (2iˆ 3 ˆj  4k)
ˆ × b,
 
then a possible value of ( a  b ). (7 ˆi  2 ˆj 3k)
ˆ is [JEE 2012, 4+3]
(A) 0 (B) 3 (C) 4 (D) 8

 
9. Let PR = 3iˆ  ˆj  2kˆ and SQ = ˆi  3jˆ  4kˆ determine diagonals of a parallelogram PQRS and

PT = ˆi  2jˆ  3kˆ be another vector. Then the volume of the parallelepiped determined by the
  
vectors PT , PQ and PS is [JEE-Advanced 2013, 2M]
(A) 5 (B) 20 (C) 10 (D) 30

Corporate Office: NAIVEDHYAM, Plot No. SP-11, Old INOX, Indra Vihar, Kota (Raj.) 324005 23
Call: 0744-2799900 Online Partner UNACADEMY
10.  
Consider the set of eight vectors V = aiˆ  bjˆ  ckˆ : a, b,c {1,1} . Three non-coplanar vectors can
be chosen from V in 2p ways. Then p is [JEE-Advanced 2013, 4, (–1)]

11. Match List – I with List – II and select the correct answer using the code given below the lists.
List – I List – II
 
P. Volume of parallelepiped determined by vectors a, b and 1. 100

c is 2. Then the volume of the parallelepiped determined by
   
     
vectors 2 a  b ,3 b  c and  c  a  is

  
Q. Volume of parallelepiped determined by vectors a, b and c 2. 30
is 5. Then the volume of the parallelepiped determined by
   
   
 
vectors 3 a  b , b  c and 2  c  a  is

R. Area of triangle with adjacent sides determined by vectors 3. 24


 
a and b is 20. Then the area of the triangle with adjacent
 
 
sides determined by vectors 2a  3b and a  b is  
S. Area of a parallelogram with adjacent sides determined by 4. 60
 
vectors a and b is 30. Then the area of the parallelogram with

 
adjacent sides determined by vectors a  b and a is

Codes :
P Q R S
(A) 4 2 3 1
(B) 2 3 1 4
(C) 3 4 1 2
(D) 1 4 3 2 [JEE-Advanced 2013, 3, (–1)]

  
12. Let x, y and z be three vectors each of magnitude
2 and the angle between each pair of them
   
is . If a is a nonzero vector perpendicular to x and y × z and b is nonzero vector
3

perpendicular to y and z  x , then [JEE (Advanced)-2014, 3]
               
(A) b  (b·z)(z x) (B) a  (a·y)(y  z) (C) a·b  (a·y)(b·z) (D) a  (a·y)(z  y)

Corporate Office: NAIVEDHYAM, Plot No. SP-11, Old INOX, Indra Vihar, Kota (Raj.) 324005 24
Call: 0744-2799900 Online Partner UNACADEMY
  
13. Let a, b and c be three non-coplanar unit vectors such that the angle between every pair of
       
them is . if a  b  b  c  pa  qb  rc, where p, q and r are scalars, then the value of
3
p  2q  r 2
2 2
is [JEE(Advanced)-2014, 3]
q2

       


14. Let PQR be a triangle. Let a = QR , b = RP and c = PQ . If | a | = 12, | b | = 4 3 and

b·c = 24, then which of the following is (are) true ?
 
| c |2  | c |2      
(A) – | a | = 12 (B)  | a | = 30 (C) | a b c a | = 48 3 (D) a.b = –72
2 2

 
15. (a) Suppose that p,q and r are three non-coplanar vectors in 3. Let the components of a
   
vector s along p,q and r be 4, 3 and 5, respectively. If the components of this vector s
        
along  p  q  r  ,  p  q  r  and  p  q  r  are x, y and z, respectively, then the value of
2x + y + z is [JEE 2015, 4,, –0M]
(b) Column-I Column-II
(A) In a triangle XYZ , let a, b and c be the length of the sides (P) 1
2 2 2
opposite to the angles X, Y and Z, respectively. If 2 (a – b ) = c
sin(X  Y)
and  = , then possible values of n for which
sin Z
cos(n) = 0 is (are)
(B) In a triangle XYZ , let a, b and c be the length of the sides (Q) 2
opposite to the angles, X Y and Z, respectively. If
1 + cos2X – 2 cos 2Y = 2 sinX sinY, then possible value(s)
a
of is (are)
b
(C) In R2, Let 3iˆ  ˆj, ˆi  3jˆ and  î +(1 – ) ˆj be the position vectors (R) 3
of X, Y and Z with respect to the origin O, respectively. If the

distance of Z from the bisector of the acute angle of OX and
 3
OY is , then possible value(s) of || is (are)
2
(D) Suppose that F() denotes the area of the region bounded (S) 5
2
by x = 0, x = 2, y = 4x and y = |x – 1| + |x – 2| + ax, where
8
  {0,1} . Then the value(s) of F() + 2 ,when  = 0 and (T) 6
3
   = 1, is (are) [JEE (Advanced) 2015]

Corporate Office: NAIVEDHYAM, Plot No. SP-11, Old INOX, Indra Vihar, Kota (Raj.) 324005 25
Call: 0744-2799900 Online Partner UNACADEMY
16. Let û = u1 î + u 2 ˆj + u 3 kˆ be a unit vector in 2 and ŵ 
6

1 ˆ ˆ

i  j  2kˆ . Given that there exist
  
ˆ ˆ  v) = 1. Which of the following statement(s)
a vector v in 3 such that | uˆ  v | = 1 and w.(u
is (are) correct ?

(A) There is exactly one choice for such v

(B) There are infinitely many choice for such v
(C) if û lies in the xy-plane then | u1 || u 2 |
(D) If û lies in the xz-plane then 2 | u1 || u 3 | [JEE(Advanced)-2016, 4(–2)]

17. Let O be the origin and let PQR be an arbitrary triangle. The point S is such that
           
OP.OQ  OR.OS = OR.OP  OQ.OS = OQ.OR  OP.OS
Then the triangle PQR has S as its [JEE (Advanced)-2017]
(A) circumcentre (B) incentre (C) centroid (D) orthocenter

PARAGRAPH-1
     
Let O be origin, and OX, OY, OZ be three unit vectors in the direction of the sides QR, RP, PQ ,
respectively of a triangle PQR. [JEE (Advanced)-2017]

 
18. OX  OY =
(A) sin (Q + R) (B) sin 2R (C) sin (P + R) (D) sin (P + Q)

19. If the triangle PQR varies. then the minimum value of


cos (P + Q) + cos (Q + R) + cos (R + P) is
5 3 5 3
(A) – (B) – (C) (D)
3 2 3 2

       
20. Let a and b be two unit vectors such that a.b  0 . For some x, y  R, let c  xa  yb  (a b) .
   
If c = 2 and the vector c is inlined at the same angle  to both a and b , then the value of
8cos2 is ____. [JEE (Advanced)-2018]

21. Consider the cube in the first with sides OP, OQ and OR of length 1, along the x-axis, y-axis
1 1 1
and z-axis, respectively, where O(0,0,0) is the origin, Let S  , ,  be the centre of the cube
2 2 2
and T be the vertex of the cube opposite to the origin O such that S lies on the diagranal OT. If
           
p  SP, q  SQ, r  SR, and t  ST, then the value of (p q)  (r  t) is ___.
[JEE (Advanced)-2018]

Corporate Office: NAIVEDHYAM, Plot No. SP-11, Old INOX, Indra Vihar, Kota (Raj.) 324005 26
Call: 0744-2799900 Online Partner UNACADEMY
22. Let L1 and L2 denote the lines
 ˆ
r  i  ( ˆi  2 ˆj  2k)
ˆ ,   and

r  (2iˆ  ˆj  2k),
ˆ 

respectively. If L3 is a line which is perpendicular to both L1 and L2 and cuts both of them, then
which of the following options describe(s) L3 ? [JEE (Advanced)-2019]
 2
(A) r  (4iˆ  ˆj  k)
ˆ  t(2iˆ  2 ˆj  k),
ˆ t R
9

(B) r  t(2iˆ  2 ˆj  k),
ˆ t 

 1
(C) r  (2 ˆi  k)
ˆ  t(2 iˆ  2 ˆj  k),
ˆ t 
3
 2
(D) r  (2iˆ  ˆj  2k)
ˆ  t(2iˆ  2 ˆj  k),
ˆ t 
9

23. Three lines



L1 : r  ˆi,  

L2 : r  kˆ  ˆj,  and

L3 : r  ˆi  ˆj  vkˆ , v
are given. For which point(s) Q and L2 can we find a point P on L1 and a point R on L3 so that
P, Q and R are collinear ? [JEE (Advanced)-2019]
1 1
(1) kˆ  ˆj (2) kˆ  ˆj (3) k̂ (4) kˆ  ˆj
2 2

    
24. Let a  2iˆ  ˆj  kˆ and b  ˆi  2ˆj  kˆ be two vectors. Consider a vector c  a  b , ,   R. If
      
the projections of c on the vector (a  b) is 3 2 , then the minimum value of (c  (a  b))  c
equals _____ [JEE (Advanced)-2019]

   
        a.(c  b) |a|
25. In a triangle PQR, let a  QR , b  RP and c  PQ . If | a |  3 , | b |  4 and       ,
c(a  b) |a||b|
 2
then the value of a  b is ______ [JEE (Advanced)-2020]

Corporate Office: NAIVEDHYAM, Plot No. SP-11, Old INOX, Indra Vihar, Kota (Raj.) 324005 27
Call: 0744-2799900 Online Partner UNACADEMY
 
26. Let a and b be positive real numbers. Suppose PQ  aiˆ  bjˆ and PS  aiˆ  bjˆ are adjacent sides
   
of a parallelogram PQRS. Let u and v be the projection vectors of w  ˆi  ˆj along PQ and
   
PS , respectively. If | u |  | v |  | w | and if the area of the parallelogram PQRS is 8, then which
of the following statements is/are TRUE ? [JEE (Advanced)-2020]
(A) a + b = 4
(B) a – b = 2
(C) The length of the diagonal PR of the parallelogram PQRS is 4
  
(D) w is an angle bisector of the vectors PQ and PS

Corporate Office: NAIVEDHYAM, Plot No. SP-11, Old INOX, Indra Vihar, Kota (Raj.) 324005 28
Call: 0744-2799900 Online Partner UNACADEMY
EXERCISE # 5
STRAIGHT OBJECTIVE TYPE

    


1. Given a parallelogram ABCD. If | AB | = a, | AD | = b and | AC | = c, then DB · AB has the
value
3a 2  b 2  c2 a 2  3b 2  c2 a 2  b 2  3c2
(A) (B) (C) (D) none
2 2 2

2. L1 and L2 are two lines whose vector equations are


L1 : 
r
 
=   cos   3 ˆi     
2 sin  ˆj  cos   3 kˆ 
 


L2 : r   aiˆ  bjˆ  ckˆ , 
where  and  are scalars and  is the acute angle between L1 and L2.
If the angle '' is independent of  then the value of '' is
   
(A) (B) (C) (D)
6 4 3 2

 
3. In the isosceles triangle ABC, | AB | = | BC | = 8, a point E divides AB internally in the ratio
  
1 : 3, then the cosine of the angle between CE and CA is (where | CA | = 12)
3 7 3 8 3 7 3 8
(A) – (B) (C) (D)
8 17 8 17

     
If p  3a  5b;q  2a  b; r = a  4b; s  a  b are four vectors such that sin  p ^ q  = 1 and
       
4.
   
 
sin  r ^ s  = 1 then cos a ^ b is :
19 19
(A)  (B) 0 (C) 1 (D)
5 43 5 43

   2


5. In a quadrilateral ABCD, AC is the bisector of the AB ^ AD which is
3
,  
    
15 | AC | = 3| AB | = 5| AD | then cos BA ^ CD is  
14 21 2 2 7
(A)  (B)  (C) (D)
7 2 7 3 7 14

Corporate Office: NAIVEDHYAM, Plot No. SP-11, Old INOX, Indra Vihar, Kota (Raj.) 324005 29
Call: 0744-2799900 Online Partner UNACADEMY
6. If the two adjacent sides of two rectangles are represented by the vectors
           
p  5a  3b;q  a  2b and r  4a  b; s  a  b respectively, then the angle between the
 1     1  
vectors x   p  r  s  and y   r  s 
3 5
 19   19 
(A) is –cos–1  5 43  (B) is cos–1  
 5 43 
 19 
(C) is – cos–1   (D) cannot be evaluated
 5 43 

7. A rigid body rotates about an axis through the origin with an angular velocity

10 3 radians/sec. If  points in the direction of ˆi  ˆj  kˆ then the equation to the locus of the
points having tangential speed 20 m/sec. is
(A) x2 + y2 + z2 – x y – y z – z x – 1 = 0
(B) x2 + y2 + z2 – 2 x y – 2 y z – 2 z x – 1 = 0
(C) x2 + y2 + z2– x y – y z – z x – 2 = 0
(D) x2 + y2 + z2– 2 x y – 2 y z – 2 z x – 2 = 0

MULTIPLE OBJECTIVE TYPE


        
8. If a, b, c be three non zero vectors satisfying the condition a  b  c and b  c  a then which of
the following always hold(s) good?
    
(A) a, b, c are orthogonal in pairs (B) a b c  = | b |
   
(C) a b c  = | c |2 (D) | b | = | c |

  
9. Given the following information about the non zero vectors A, B and C
         

(i) A  B  A  0 (ii) B · B = 4 (iii) A . B = –6 (iv) B·C  6

Which one of the following holds good?


         
(A) A  B  0 
(B) A· B  C  0  (C) A·A  8 (D) A·C  9
   
10. If A, B,C and D are four non zero vectors in the same plane no two of which are collinear then
which of the following hold(s) good?
       
(A) ( A  B )·( C  D ) = 0 (B) ( A  C )·( B  D )0
         
(C) ( A  B ) × ( C  D ) = 0 (D) ( A  C ) × ( B  D )  0

Corporate Office: NAIVEDHYAM, Plot No. SP-11, Old INOX, Indra Vihar, Kota (Raj.) 324005 30
Call: 0744-2799900 Online Partner UNACADEMY
   
11. If a, b, c & d are the pv's of the points A, B, C & D respectively in three dimensional space &
   
satisfy the relation 3 a – 2 b + c – 2 d = 0, then :
(A) A, B, C & D are coplanar
(B) the line joining the points B & D divides the line joining the point A & C in the ratio 2 : 1.
(C) the line joining the points A & C divides the line joining the points B & D in the ratio 1 : 1
   
(D) the four vectors a, b, c & d are linearly dependent.

6  2 3 
        
12. The vectors u =  3 ; v =  6  ; w =  2 
 2  3   6 
(A) form a left handed system
(B) form a right handed system
(C) are linearly independent
(D) are such that each is perpendicular to the plane containing the other two.

  
13. If a, b, c are non-zero, non-collinear vectors such that a vector
     

   
p  ab cos 2  a ^ b c and a vector q  accos     a ^ c   b then p  q is
 
 
(A) parallel to a (B) perpendicular to a
   
(C) coplanar with b & c (D) coplanar with a and c

14. Which of the following statement(s) hold good ?


    
(A) if a·b  a·c  b  c(a  0)
     
(B) if a  b  a  c  b  c(a  0)
        
(C) if a·b  a·c and a  b  a  c  b  c (a  0)
     
   v v v v
(D) if v1 , v2 , v3 are non coplanar vectors and k1   2 3 ; k 2   3 1
v1·(v 2  v3 ) v1·(v 2  v3 )
     
v v 1
and k 3   1 2 then k1.(k 2  k 3 )    
v1·(v 2  v3 ) v1.(v2  v3 )

 ˆ makes angles , ,  with xy, yz and zx planes


15. If the line r  2iˆ  ˆj  3kˆ  (iˆ ˆj 2 k)
respectively then which of the following are not possible?
(A) sin2 + sin2 + sin2 = 2 & cos2 + cos2 + cos2 = 1
(B) tan2 + tan2 + tan2 = 7 & cot2 + cot2 + cot2 = 5/3
(C) sin2 + sin2 + sin2 = 1 & cos2 + cos2 + cos2 = 2
(D) sec2 + sec2 + sec2 = 10 & cosec2 + cosec2 + cosec2 = 14/3

Corporate Office: NAIVEDHYAM, Plot No. SP-11, Old INOX, Indra Vihar, Kota (Raj.) 324005 31
Call: 0744-2799900 Online Partner UNACADEMY
16. If a, b, c are different real numbers and a ˆi  bjˆ  ck;bi
ˆ ˆ  cjˆ  akˆ & ciˆ ajˆ  bkˆ are position vectors
of three non-collinear points A, B & C then :

(A) centroid of triangle ABC is


a bc ˆ ˆ ˆ
3

i  j k 
(B) ˆi  ˆj  kˆ is equally inclined to the three vectors
(C) perpendicular from the origin to the plane of triangle ABC meet at centroid
(D) triangle ABC is an equilateral triangle.

17. A vector of magnitude 10 along the normal to the curve 3x2 + 8xy + 2y2 – 3 = 0 at its point
P(1, 0) can be
(A) 6iˆ  8jˆ (B) 6iˆ  8jˆ (C) 6iˆ  8jˆ (D) 6iˆ  8jˆ

18. Let OAB be a regular triangle with side length unity (O being the origin). Also M,N are the
points of trisection of AB,M being closer to A and N closer to B. Position vectors of A,B,M
  
and N are a, b, m and n respectively. Which of the following hold(s) good ?
   2 1    5 1
(A) m  xa  yb  and y = (B) m  xa  yb  and y =
3 3 6 6
 13  15
(C) m.n equals (D) m.n equals
18 18

19. If A( a ); B( b ); C( c ) and D( d ) are four points such that


a = 2iˆ  4jˆ  3k;
ˆ b  2iˆ  8j; ˆ d  4iˆ  ˆj  7kˆ
ˆ c  ˆi  3jˆ  5k;
d is the shortest distance between the lines AB and CD, then which of the following is True?
  
 AB CD BD 
(A) d = 0, hence AB and CD intersect (B) d =    
| AB  CD |
  
 AB CD AC 
(D) d =    
23
(C) AB and CD are skew lines and d =
13 | AB  CD |

20. Which of the following statement(s) is(are) incorrect ?


     
(A) The relation | (u v) || u.v | is only possible if atleast one of the vectors u and v is null
vector.

(B) Every vector contained in the line r(t) = 1  2t,1  3t,1  4t is parallel to the vector 1,1,1 .
     
(C) If scalar triple product of three vectors, u, v, w is larger than | u v | then | w | > 1 .
(D) The distance between the x-axis and the line x = y = 1 is 2 .

  
21. Given three vectors U  2iˆ  3jˆ  6k;
ˆ V  6iˆ  2ˆj  3k;
ˆ W  3iˆ  6ˆj  2kˆ

Corporate Office: NAIVEDHYAM, Plot No. SP-11, Old INOX, Indra Vihar, Kota (Raj.) 324005 32
Call: 0744-2799900 Online Partner UNACADEMY
  
Which of the following hold good for the vectors U , V and W ?
  
(A) U , V and W are linearly dependent
   
(B) ( U × V ) × W = 0
  
(C) U , V and W form a triplet of mutually perpendicular vectors
   
(D) U ×( V × W ) = 0

22. Which of the following statement(s) is/are true in respect of the lines
      
r  a  b; r  c  d where b  d  0


–1 | b·d |

(A) acute angle between the lines is cos    
 | b || d | 
    
(B) The lines would intersect if [c b d] = [a b d]
 
(C) The lines will be skew if [c a b d]  0
 
(D) If the lines intersect at r  r0 , then the equation of the plane containing the lines is
  
[r  r0 b d]  0

 
23. Let a and b be two non-zero and non-collinear vectors then which of the following is/are
always correct?
     ˆ ˆ
(A) a  b  [a b ˆi]iˆ [a b ˆj]  [a b k]k
  ˆ  ˆ  ˆ  ˆ  ˆ  ˆ
(B) a.b  (a.i)(b.i)  (a. j)(b. j)  (a.k)(b.k)
 ˆ band
ˆ   
(C) if u  aˆ  (aˆ·b) v  aˆ  bˆ then| u || v |
          
(D) if c  a  (a  b)and d  b  (a b) then c  d  0

COMPREHENSION TYPE

Paragraph for questions nos. 24 to 26


   
Consider three vectors p  ˆi  ˆj  k,
ˆ q  2iˆ  4jˆ  kˆ and r  ˆi  ˆj  3kˆ and let s be a unit vector,
then
  
24. p, q and r are
(A) linearly dependent
(B) can form the sides of a possible triangle
  
(C) such that the vectors (q  r) is orthogonal to p
(D) such that each one of these can be expressed as a linear combination of the other two

     
25. If (p  q) × r = up  vq  wr , then (u + v + w) equals to

Corporate Office: NAIVEDHYAM, Plot No. SP-11, Old INOX, Indra Vihar, Kota (Raj.) 324005 33
Call: 0744-2799900 Online Partner UNACADEMY
(A) 8 (B) 2 (C) – 2 (D) 4

        
26. The magnitude of the vector (p·s)(q r)  (q·s)(r p)  (r·s)(p q) is
(A) 4 (B) 8 (C) 18 (D) 2

MATRIX MATCH TYPE


27. Column-I Column-II

(A) P is point in the plane of the triangle ABC. (P) centroid


  
The pv’s of A,B and C are a, b and c respectively with
   
  
respect to P as the origin. If b  c · b  c = 0 and
   
 c  a · c  a  = 0, then w.r.t. the triangle ABC,P is its
  
(B) If a,b,c are the position vectors of the three non collinear (Q) orthocentre
points A,B and C respectively such that the vector

V  PA  PB  PC is a null vector then w.r.t. the ABC, P is
its

(C) If P is a point inside the ABC such that the vector (R) Incentre
   
   
R  (BC)PA  (CA) PB  (AB) PC is a null vector then
w.r.t. the ABC, P is its

(D) If P is a point in the plane of the triangle ABC such that the (S) circumcentre
   
scalar product PA.CB and PB.AC vanishes, then w.r.t. the
ABC, P is its

Corporate Office: NAIVEDHYAM, Plot No. SP-11, Old INOX, Indra Vihar, Kota (Raj.) 324005 34
Call: 0744-2799900 Online Partner UNACADEMY
EXERCISE # 6
1. Given a tetrahedron D-ABC with AB = 12 , CD = 6. If the shortest distance between the skew

lines AB and CD is 8 and the angle between them is , then find the volume of tetrahedron.
6


2. A vector V = v1ˆi  v2ˆj  v3kˆ satisfies the following conditions :

(i) magnitude of V is 7 2

(ii) V is parallel to the plane x – 2y + z = 6
 
(iii) V is orthogonal to the vector 2iˆ  3jˆ  6ˆj and (iv) V . î > 0
Find the value of (v1 + v2 + v3).

         
3. Let (p q) × r + (q·r) q = (x 2  y2 ) q + (14 – 4x – 6y) p and ( r·r ) p = r where p and q are
two non-zero non-collinear vectors and x and y are scalars. Find the value of (x + y).

AE AF
4. In a ABC, points E and F divide sides AC and AB respectively so that = 4 and = 1.
EC FB
Suppose D is a point on side BC. Let G be the intersection of EF and AD and suppose D is
AG 3 BD a
situated so that = . If the ratio = , where a and b are in their lowest form, find the
GD 2 DC b
value of (a + b).


5. Let u be a vector on rectangular coordinate system with sloping angle 60°. Suppose that | u  ˆi |
 
is geometric mean of | u | and | u  2 ˆi | where î is the unit vector along x-axis then find the value

of | u | .

   
6. a, b, c and d are the position vectors of the points A  (x, y, z);B  (y, –2z, 3x) ;C  (2z, 3x,–y)
      

   

and D  (1, –1, 2) respectively. If | a | = 2 3 ; a ^ b =  a ^ c  ; a ^ d = and a ^ ˆj is
2
 
obtuse, then find x, y, z.

7. The length of the edge of the regular tetrahedron D–ABC is 'a'. Point E and F are taken on the
 
edges AD and BD respectively such that E divides DA and F divides BD in the ratio 2 : 1
each. Then find the area of triangle CEF.

8. The position vectors of the points A, B, C are respectively (1, 1, 1) ; (1, –1, 2) ; (0, 2, –1). Find
a unit vector parallel to the plane determined by ABC &perpendicular to the vector(1,0,1).

Corporate Office: NAIVEDHYAM, Plot No. SP-11, Old INOX, Indra Vihar, Kota (Raj.) 324005 35
Call: 0744-2799900 Online Partner UNACADEMY
9. The position vectors of the vertices A,B and C of a tetrahedron are (1,1,1), (1,0,0) and (3,0,0)
respectively. The altitude from the vertex D to the opposite face ABC meets the median line
through A of the triangle ABC at a point E. If the length of side AD is 4 and volume of the
tetrahedron is 2 2 / 3 then find the all possible position vectors of the point E.

10. Given non zero number x1, x2, x3 ; y1, y2, y3 and z1, z2 and z3
(i) Can the given numbers satisfy
x1 x 2 x 3  x1x 2  y1 y 2  z1z 2  0

y1 y 2 y3 = 0 and  x 2 x 3  y 2 y3  z 2 z 3  0
x x y y z z 0
z1 z 2 z3  3 1 3 1 3 1
(ii) If xi > 0 and yi < 0 for all i = 1, 2, 3 and P (x1, x2, x3); Q(y1, y2, y3) and O(0, 0, 0) can the
triangle POQ be a right angled triangle ?

        
11. Given that a, b, p, q are four vectors such that a  b =  p , b·q = 0 and (b)2 = 1, where  is a
    
scalar then prove that | (a·q) p (p·q)a | = | p·q | .
2
â bˆ

12. Let g() =  (2 t  1) dt, where  is the angle between â and b̂ .If volume of the parallelopiped
 
2
 â.bˆ

ˆ aˆ  bˆ and aˆ  aˆ  bˆ (where angle


whose coterminous edges are represented by vectors a,  
p
between â and b̂ is taken from the equation 2g() – 1 = 0), is then find the least value of (p + q).
q

13. (a) Find a unit vector â which makes an angle (/4) with axis of z & is such that â + î + ˆj is a
unit vector.
  
 3 | a b |  
(b) If a and b are any two unit vectors, then find the range of  2 | a b | .
2
      
14. Given four non zero vectors a, b, c and d . The vectors a, b and c are coplanar but not collinear
   
pair by pair and vector d is not coplanar with vectors a, b and c and
          
       
a ^ b = b ^ c = , d ^ a = , d ^ b = , then prove that d ^ c = cos–1(cso – cos)
3
 
15. Given three points on the xy plane on O(0, 0), A(1, 0) and B(–1, 0). Point P is moving on the
   
   
plane satisfying the condition PA·PB + 3 OA·OB = 0. If the maximum and minimum
 
values of | PA || PB | are M and m respectively then find the values of M2 + m2.

           
16. Let a, b, c are unit vectors where | a  b |2  | b c |2  | c a |2 = 3, then | a  2 b 3c |2 is equal to

Corporate Office: NAIVEDHYAM, Plot No. SP-11, Old INOX, Indra Vihar, Kota (Raj.) 324005 36
Call: 0744-2799900 Online Partner UNACADEMY
ANSWER KEY
EXERCISE # 1
1. B 2. A 3. B 4. B 5. D 6. B 7. D
8. B 9. D 10. B 11. D 12. C 13. D 14. B
15. C 16. D 17. C 18. A 19. A 20. (i) D, (ii) B, (iii) B
21. D 22. A 23. D 24. D 25. B 26. C 27. B
28. B 29. D 30. A 31. A 32. B 33. B 34. A
35. C 36. A 37. C 38. B 39. A 40. C 41. C
42. D 43. D 44. C 45. D 46. B 47. A 48. D
49. C 50. C 51. D 52. A 53. D 54. A 55. C
56. C 57. A 58. D 59. A 60. C 61. B 62. A
63. D 64. D 65. B
66. (A) T; (B) U ; (C) P; (D) R ; (E) Q; (F) S; (G) W; (H) V

EXERCISE # 2
1
1. (9, 7) 2.  (iˆ  ˆj) 4. 13 5. 3 6. 4950 7. 7
2
8. 1125 9. x = 2, y = –1 10. (b) externally in the ratio 1 : 3
11. (i) parallel (ii) the lines intersect at the point p. v. –2 î + 2ˆj
(iii) lines are skew
1 
13. (a) cot–1(0) ; (b) cot–1 ; (c) cot–1 2 14.
3 2

 1 2  3
15.  ,  16. 3 17. (a) , (b) 51
 3 3 2
  
18. (a) 2, (b) –1, (c) –12 19. 101 20. F= 2a1  5a 2  3a 3

4 ˆ 1 ˆ 1 ˆ 6 3
21. i j k 22. (i) 14 (ii) 6 (iii) 10 (iv) 6
2 2 2 7 5
         
 a  (c.a) c  b c b  (c.b) c  a  c
23. 6 24. 13 25. x  ,y = 
1  c2 1  c2
26. 75 27. 488

Corporate Office: NAIVEDHYAM, Plot No. SP-11, Old INOX, Indra Vihar, Kota (Raj.) 324005 37
Call: 0744-2799900 Online Partner UNACADEMY
EXERCISE # 3 (JM)
1. 3 2. 4 3. 4 4. 4 5. 3 6. 1 7. 2
8. 3 9. 4 10. 4 11. 3 12. 3 13. 1 14. 1
15. 4 16. 3 17. 1 18. 3 19. 2 20. 3 21. 3
22. 1 23. 3 24. 4 25. 3 26. 2 27. 2 28. 3
29. 1 30. 3 31. 3 32. 3 33. 1 34. 3 35. 3
36 1 37. Bonus 38. 1 39. 4.00 40. 3 41. 4 42. 1.00
43. 30.00 44. 2.00 45. 2 46. 1 47. 2 48. 18.00 49. 2
50. 6.00 51. 4.00 52. 1.00

EXERCISE # 4 (JA)
1. ˆ  vˆ  2(a.v)a
w ˆˆ ˆ 2. (a) A (b) B
3. (a) C (b) B (c) C 4. (a) A (b) A
5. (a) C (b) (A) Q,S; (B) P,R,S,T; (C) T, (D) R
6. (a) B (b) 5
7. (a) C (b) A,D (c) 9 8. (a) 3 (b) C
9. C 10. 5 11. C 12. A,B,C 13. 4 14. A,C,D
15. (a) Bonus, (b) (AP,R,S); (B P); (C P,Q); ( D S; T)
16. B,C 17. D 18. D 19. B 20. 3.00 21. 0.50
22. A,C,D 23. A,D 24. 18.00 25. 108.00 26. A,C

EXERCISE # 5
1. A 2. A 3. C 4. D 5. C 6. B 7. C
8. A,C 9. A,B,D 10. B,C 11. A,C,D 12. A,C,D 13. B,C 14. C,D
15. A,B,D 16. A,B,C,D 17. A,D 18. A,C 19. B,C,D 20. A,B,D
21. B,C,D 22. A,B,C,D 23. A,B,C 24. C 25. B 26. A
27. (A) S; (B) P; (C) R; (D) Q

EXERCISE # 6
1. 48 2. 12 3. 5 4. 9 5. 2 – 1 6. x = 2, y = –2, z = –2

 ˆi  5jˆ  kˆ 
2
5a 1
7. sq. units 8. ± 9. (–1, 3, 3) and (3, –1, –1)
12 3 3 3
1 ˆ 1 ˆ 1 ˆ
10. NO, NO 12. 5 13. (a) i  j k, (c) Range : [3, 5]
2 2 2
15. 34 16. 19

Corporate Office: NAIVEDHYAM, Plot No. SP-11, Old INOX, Indra Vihar, Kota (Raj.) 324005 38
Call: 0744-2799900 Online Partner UNACADEMY
3D-COORDINATE GEOMETRY
EXERCISE # 1
STRAIGHT OBJECTIVE TYPE
  ˆ and r  ˆi  ˆj  3k,   
1. Consider three vectors p  ˆi  ˆj  k,
ˆ q  2iˆ  4ˆj  k, ˆ . If p,q and r denotes the
position vector of three non-collinear points then the equation of the plane containing these
points is
(A) 2x – 3y + 1 = 0 (B) x – 3y + 2z = 0 (C) 3x – y + z – 3 = 0 (D) 3x – y – 2 = 0

2. The intercept made by the plane r.n  q on the x-axis is

(A) 
q
î.n
(B)
î.n
q

(C) î.n q  q
(D) 
|n|

3. If the distance between the planes


8x + 12y – 14z = 2
and 4x + 6y – 7z = 2
1 N(N  1)
can be expressed in the form where N is natural then the value of is
N 2
(A) 4950 (B) 5050 (C) 5150 (D) 5151

4. A plane passes through the point P(4, 0, 0) and Q(0, 0, 4) and is parallel to the y-axis. The
distance of the plane from the origin is
(A) 2 (B) 4 (C) 2 (D) 2 2

5. If from the point P (f, g, h) perpendiculars PL, PM be drawn to yz and zx planes then the
equation to the plane OLM is
x y z x y z x y z x y z
(A)   = 0 (B)    0 (C)    0 (D)     0
f g h f g h f g h f g h

6. If the plane 2x – 3y + 6z – 11 = 0 makes an angle sin–1(k) with x-axis, then k is equal to


(A) 3 /2 (B) 2/7 (C) 2 /3 (D) 1

7. The plane XOZ divides the join of (1, –1, 5) and (2, 3, 4) in the ratio  : 1 , then  is
(A) – 3 (B) – 1/3 (C) 3 (D) 1/3

8. A variable plane forms a tetrahedron of constant volume 64 K3 with the coordinate planes and
the origin, then locus of the centroid of the tetrahedron is
(A) x3 + y3 + z3 = 6K3 (B) xyz = 6K3
(C) x2 + y2 + z2 = 4K2 (D) x–2 + y–2 + z–2 = 4K–2

Corporate Office: NAIVEDHYAM, Plot No. SP-11, Old INOX, Indra Vihar, Kota (Raj.) 324005 39
Call: 0744-2799900 Online Partner UNACADEMY
9. Let ABCD be a tetrahedron such that the edges AB, AC and AD are mutually perpendicular.
Let the area of triangles ABC, ACD and ADB be 3, 4 and 5 sq. units respectively. Then the
area of the triangle BCD, is
(A) 5 2 (B) 5 (C) 5/ 2 (D) 5/2

10. Equation of the line which passes through the point with p. v. (2, 1, 0) and perpendicular to the
plane containing the vectors ˆi  ˆj and ˆj  kˆ is
 
(A) r = (2, 1, 0) + t (1, –1, 1) (B) r = (2, 1, 0) + t (–1, 1, 1)
 
(C) r = (2, 1, 0) + t (1, 1, –1) (D) r = (2, 1, 0) + t (1, 1, 1)
where t is a parameter

11. Which of the following planes are parallel but not identical?
P1 : 4x – 2y + 6z = 3
P2 : 4x – 2y – 2z = 6
P3 : –6x + 3y – 9z = 5
P4 : 2x – y – z = 3
(A) P2 & P3 (B) P2 & P4 (C) P1 & P3 (D) P1 & P4

12. A parallelopiped is formed by planes drawn through the points (1, 2, 3) and (9, 8, 5) parallel to
the coordinate planes then which of the following is not the length of an edge of this
rectangular parallelopiped
(A) 2 (B) 4 (C) 6 (D) 8

 ˆ + (iˆ 2 ˆj 3k)


ˆ in the scalar dot product form is
13. Vector equation of the plane r  ˆi  ˆj  (iˆ ˆj k)
 
(A) r .(5iˆ 2 ˆj 3k)
ˆ 7 (B) r .(5iˆ 2 ˆj 3k)
ˆ 7
 
(C) r .(5iˆ 2 ˆj 3k)
ˆ 7 (D) r .(5iˆ 2 ˆj 3k)
ˆ 7

14. The vector equations of the two lines L1 and L2 are given by
 
L1 : r  2 ˆi  9 ˆj 13kˆ  (iˆ 2jˆ  3k)
ˆ ; L2 : r  3iˆ 7 ˆj pkˆ  ( ˆi  2jˆ  3k)
ˆ
then the lines L1 and L2 are
(A) skew lines for all pR
(B) intersecting for all pR and the point of intersection is (–1, 3, 4)
(C) intersecting lines for p = – 2
(D) intersecting for all real pR

15. Consider the plane (x, y, z) = (0, 1, 1) + (1, – 1, 1) + (2, – 1, 0). The distance of this plane
from the origin is
(A) 1/3 (B) 3 /2 (C) 3 / 2 (D) 2/ 3

Corporate Office: NAIVEDHYAM, Plot No. SP-11, Old INOX, Indra Vihar, Kota (Raj.) 324005 40
Call: 0744-2799900 Online Partner UNACADEMY
x2 y9 z  13 xa y7 z2
16. The value of 'a' for which the lines = = and = =
1 2 3 1 2 3
intersect, is
(A) – 5 (B) – 2 (C) 5 (D) – 3

17. Given A (1, –1, 0) ; B(3, 1, 2) ; C(2, –2, 4) and D(–1, 1, –1) which of the following points
neither lie on AB nor on CD?
(A) (2, 2, 4) (B) (2, –2, 4) (C) (2, 0,1) (D) (0, –2, –1)

x 1 y2 z3
18. For the line = = , which one of the following is incorrect?
1 2 3
x y z
(A) it lies in the plane x – 2y + z = 0 (B) it is same as line  
1 2 3
(C) it passes through (2, 3, 5) (D) it is parallel to the plane x – 2y + z – 6 = 0

19. Given planes P1 : cy + bz = x


P2 : az + cx = y
P3 : bx + ay = z
P1, P2 and P3 pass through one line, if
(A) a2 + b2 + c2 = ab + bc + ca (B) a2 + b2 + c2 + 2abc = 1
(C) a2 + b2 + c2 = 1 (D) a2 + b2 + c2 + 2ab + 2bc + 2ca + 2abc = 1

x  x1 y  y1 z  z1
20. The line = = is
0 1 2
(A) parallel to x-axis (B) perpendicular to x-axis
(C) perpendicular to YOZ plane (D) parallel to y-axis

x 2 y3 z 4 x 1 y  4 z  5
21. The lines   and   are coplanar if
1 1 k k 2 1
(A) k = 0 or – 1 (B) k = 1 or – 1 (C) k = 0 or – 3 (D) k = 3 or – 3

22. The line which contains all points (x, y, z) which are of the form (x, y, z) = (2, –2, 5) + (1, –3, 2)
intersects the plane 2x – 3y + 4z = 163 at P and intersects the YZ plane at Q. If the distance PQ
is a b where a, b  N and a > 3 then (a + b) equals
(A) 23 (B) 95 (C) 27 (D) none

 
23. Let L1 be the line r1  2iˆ  ˆj  kˆ  (iˆ 2k)
ˆ and let L2 be the line r  3iˆ  ˆj  (iˆ ˆj  k)
2
ˆ .
Let  be the plane which contains the line L1 and is parallel to L2. The distance of the plane 
from the origin is
(A) 1/7 (B) 2/7 (C) 6 (D) none

Corporate Office: NAIVEDHYAM, Plot No. SP-11, Old INOX, Indra Vihar, Kota (Raj.) 324005 41
Call: 0744-2799900 Online Partner UNACADEMY
24. The value of m for which straight line 3x – 2y + z + 3 = 0 = 4x – 3y + 4z + 1 is parallel to the
plane 2x – y + mz – 2 = 0 is
(A) –2 (B) 8 (C) – 18 (D) 11

25. The distance of the point (–1, –5, – 10) from the point of intersection of the line
x  2 y 1 z  2
  and the plane x – y + z = 5 is
2 4 12
(A) 2 11 (B) 126 (C) 13 (D) 14

  
26. P( p ) and Q( q ) are the position vectors of two fixed points and R( r ) is the position vector of a
   
variable point. If R moves such that ( r  p )× ( r  q ) = 0 then the locus of R is
 
(A) a plane containing the origin 'O' and parallel to two non collinear vectors OP and OQ
(B) the surface of a sphere described on PQ as its diameter.
(C) a line passing through the points P and Q
(D) a set of lines parallel to the line PQ.

MATRIX MATCH TYPE

27. Consider the following four pairs of lines in column-I and match them with one or more entries
in column-II.
Column-I Column-II
(A) L1 : x = 1 + t, y = t, z = 2 – 5t (P) non coplanar lines

L2 : r = (2, 1, –3) + (2, 2, –10)
x 1 y  3 z  2
(B) L1 =   (Q) lines lie in a unique plane
2 2 1
x 2 y6 z2
L2 :  
1 1 3
(C) L1 : x = – 6t, y = 1 + 9t, z = – 3t (R) infinite planes containing both the lines
L2 : x = 1 + 2s, y = 4 – 3s, z = s
x y 1 z  2
(D) L1 :   (S) lines are not intersecting at a unique point
1 2 3
x  3 y  2 z 1
L2 :  
4 3 2

Corporate Office: NAIVEDHYAM, Plot No. SP-11, Old INOX, Indra Vihar, Kota (Raj.) 324005 42
Call: 0744-2799900 Online Partner UNACADEMY
EXERCISE # 2
1. Find the angle between the two straight lines whose direction cosines , m, n are given by

2 + 2m – n = 0 and mn + n + m = 0.

2. The plane denoted by 1 : 4x + 7y + 4z + 81 = 0 is rotated through a right angle about its line
of intersection with the plane 2 : 5x + 3y + 10z = 25. If the plane in its new position be
denoted by , and the distance of this plane from the origin is k where k  N, then find k.

3. Find the equations of the straight line passing through the point (1,2,3) to intersect the straight
line x + 1 = 2(y – 2) = z + 4 and parallel to the plane x + 5y + 4z = 0.

4. A variable plane is at a constant distance p from the origin and meets the coordinate axes in
points A,B and C respectively. Through these points, planes are drawn parallel to the
coordinates planes. Find the locus of their point of intersection.

x 1 yp z2 x y7 z7


5. Find the value of p so that the lines = = and = = are in the
3 2 1 1 3 2
same plane, for this value of p, find the coordinates of their point of intersection and the
equation of the plane containing them.

6. Find the equations to the line of greatest slope through the point (7, 2, –1) in the plane x–2y + 3z = 0
assuming that the axes are so placed that the plane 2x + 3y – 4z = 0 is horizontal.

2   1

7. Let L be the line given by r =  2     0  and let P be the point (2,–1,1). Also suppose that
   
 1  1 
E be the plane containing three non collinear points A(0,1,1) ; B(1,2,2) and C(1,0,1).
Find
(a) Distance between the point P and the line L.
(b) Equation of the plane E.
(c) Equation the plane F containing the line L and the point P.
(d) Acute angle between the plane E and F.
(e) Volume of the parallelopiped by A,B,C and the point D(–3,0,1).

8. The position vectors of the four angular points of a tetrahedron OABC are (0, 0, 0); (0, 0, 2);
(0, 4, 0) and (6, 0, 0) respectively. A point P inside the tetrahedron is at the same distance 'r'
from the four plane faces of the tetrahedron. Find the value of 'r'.

Corporate Office: NAIVEDHYAM, Plot No. SP-11, Old INOX, Indra Vihar, Kota (Raj.) 324005 43
Call: 0744-2799900 Online Partner UNACADEMY
9. Let the equation of the plane containing the line x – y – z – 4 = 0 = x + y + 2z – 4 and is
parallel to the line of intersection of the planes 2x + 3y + z = 1 and x + 3y + 2z = 2 be
x + Ay + Bz + C = 0 Compute the value of |A + B + C|.

x 1 y2 z3
10. Find the equation of the line which is reflection of the line = = in the plane
9 1 3
3x – 3y + 10z = 26.

x 1 y z
11. Find the equation of the plane containing the line   and parallel to the line
2 3 2
x 3 y z2
 = .Find also the S.D. between the two lines.
2 5 4

12. Consider the plane


 1 1  1 

E : r = 1     2    0
     
1  0  1 
Let F be the plane containing the point A(–4,2,2) and parallel to E.
Suppose the point B is on the plane E such that B has a minimum distance from the point A. If
C(–3,0,4) lies in the plane F. Find the area of the angle ABC.

13. The equation of the plane which has the property that the point Q(5,4,5) is the reflection of
point P(1,2,3) through that plane, is ax + by + cz = d where a,b,c,d  N. Find the least value of
(a + b + c + d).

14. Find the equation of the line passing through the point (4,–14,4) and intersecting the line of
intersection of the planes : 3x + 2y – z = 5 and x – 2y – 2z = –1 at right angles.

15. Find the point where the line of intersection of the planes x – 2y + z = 1 and x + 2y – 2z = 5,
intersects the plane 2x + 2y + z + 6 = 0.

16. Feet of the perpendicular drawn from the point P(2,3,–5) on the axes of coordinates are A,B
and C. Find the equation of the plane passing through their feet and the area of ABC.

x 1 y2 z
17. Find the equation of the plane containing the straight line = = and perpendicular
2 3 5
to the plane x – y + z + 2 = 0.

Corporate Office: NAIVEDHYAM, Plot No. SP-11, Old INOX, Indra Vihar, Kota (Raj.) 324005 44
Call: 0744-2799900 Online Partner UNACADEMY
EXERCISE # 3 (JM)
1. The two lines x = ay + b, z = cy + d; and x = a' y + b; z = c' y + d' are perpendicular to each
other if- [AIEEE-2006]
a c a c
(1) a a' + cc' = 1 (2)  = –1 (3)  =1 (4) aa' + cc' = –1
a ' c' a ' c'

2. The image of the point (–1, 3, 4) in the plane x – 2y = 0 is- [AIEEE-2006]


 17 19 
(1) (15, 11, 4) (2)   ,  ,1 (3) (8, 4, 4) (4) None of these
 3 3 


3. If a line makes an angle of with the positive directions of each of x-axis and y-axis, then the
4
angle that the line makes with the positive direction of the z-axis is- [AIEEE-2007]
(1) /6 (2) /3 (3) /4 (4) /2

4. If (2, 3, 5) is one end of a diameter of the sphere x2 + y2 + z2 – 6x – 12y – 2z + 20 = 0, then the


coordinates of the other end of the diameter are- [AIEEE-2007]
(1) (4, 9, –3) (2) (4, –3, 3) (3) (4, 3, 5) (4) (4, 3, –3)

5. The line passing through the points (5, 1, a) and (3, b, 1) crosses the yz-plane at the point
 17 13 
 0, ,  .Then- [AIEEE-2008]
 2 2 
(1) a = 2, b = 8 (2) a = 4, b = 6 (3) a = 6, b = 4 (4) a = 8, b = 2

x2 y 1 z  2
6. Let the line = = lie in the plane x + 3y – z +  = 0. Then (, ) equals
3 5 2
[AIEEE-2009]
(1) (5, – 15) (2) (–5, 5) (3) (6, –17) (4) (–6, 7)

7. The projections of a vector on the three coordinate axis are 6, –3, 2 respectively. The direction
cosines of the vector are :- [AIEEE-2009]
6 3 2 6 3 2 6 3 2
(A) , , (B) , , (C) 6, –3, 2 (D) , ,
7 7 7 7 7 7 5 5 5

8. Statement–1 : The point A(3, 1, 6) is the mirror image of the point B(1, 3, 4) in the plane
x – y + z = 5. [AIEEE-2010]
Statement–2 : The plane x – y + z = 5 bisects the line segment joining A(3, 1, 6) and B(1, 3, 4).
(1) Statement–1 is true, Statement–2 is true; Statement–2 is a correct explanation for
Statement–1.
(2) Statement–1 is true, Statement–2 is true ; Statement–2 is not a correct explanation for
statement–1.
(3) Statement–1 is true, Statement–2 is false.
(4) Statement–1 is false, Statement–2 is true.

Corporate Office: NAIVEDHYAM, Plot No. SP-11, Old INOX, Indra Vihar, Kota (Raj.) 324005 45
Call: 0744-2799900 Online Partner UNACADEMY
y 1 z  3  5 
 and the plane x + 2y + 3z = 4 is cos–1 
 14 
9. If the angle between the line x = ,
2   
then  equals:- [AIEEE-2011]
2 5 2 3
(1) (2) (3) (4)
5 3 3 2

10. Statement-1 : The point A(1, 0, 7) is the mirror image of the point B(1, 6, 3) in the line :
x y 1 z  2
  .
1 2 3
x y 1 z  2
Statement-2 : The line :   bisects the line segment joining A (1, 0, 7) and B
1 2 3
(1, 6, 3). [AIEEE-2011]
(1) Statement-1 is true, Statement-2 is false.
(2) Statement-1 is false, Statement-2 is true
(3) Statement-1 is true, Statement-2 is true; Statement-2 is a correct explanation for Statement-1
(4) Statement-1 is true, Statement-2 is true; Statement-2 is not a correct explanation for
Statement-1.

11. The distance of the point (1, –5, 9) from the plane x – y + z = 5 measured along a straight line
x = y = z is : [AIEEE-2011]
(1) 3 5 (2) 10 3 (3) 5 3 (4) 3 10

12. An equation of a plane parallel to the plane x – 2y + 2z – 5 = 0 and at a unit distance from the
origin is : [AIEEE-2012]
(1) x – 2y + 2z + 5 = 0 (2) x – 2y + 2z – 3 = 0
(3) x – 2y + 2z + 1 = 0 (4) x – 2y + 2z – 1 = 0

x 1 y  1 z 1 x 3 yk z
13. If the lines   and   intersect, then k is equal to :
2 3 4 1 2 1
2 9
(1) 0 (2) – 1 (3) (4) [AIEEE-2012]
9 2

14. Distance between two parallel planes 2x + y + 2z = 8 and 4x + 2y + 4z + 5 = 0 is :-

3 5 7 9
(1) (2) (3) (4) [JEE-MAIN 2013]
2 2 2 2

x 2 y3 z 4 x 1 y  4 z  5
15. If the lines   and   are coplanar, then k can have :
1 1 k k 2 1
(1) any value (2) exactly one value [JEE-MAIN 2013]
(3) exactly two values (4) exactly three values.

Corporate Office: NAIVEDHYAM, Plot No. SP-11, Old INOX, Indra Vihar, Kota (Raj.) 324005 46
Call: 0744-2799900 Online Partner UNACADEMY
 
16. A vector n is inclined to x-axis at 45º, to y-axis at 60º and at an acute angle to z-axis. If n is a
normal to a plane passing through the point ( 2 , – 1, 1), then the equation of the plane is :
(1) 2 x – y – z = 2 (2) 2 x + y + z = 2 [JEE-MAIN Online 2013]
(3) 3 2 x – 4y – 3z = 7 (4) 4 2 x + 7y + z = 2

17. The acute angle between two lines such that the direction cosines , m, n of each of them

satisfy the equations  + m + n = 0 and 2 + m2 – n2 = 0 is :- [JEE-MAIN Online 2013]


(1) 30° (2) 45° (3) 60° (4) 15°

18. Let Q be the foot of perpendicular from the origin to the plane 4x – 3y + z + 13 = 0 and R be a
point (–1, 1, –6) on the plane. Then length QR is :- [JEE-MAIN Online 2013]
7 19 3
(1) 3 (2) 14 (3) (4)
2 2 2

19. If the projections of a line segment on the x, y and z-axes in 3-dimensional space are 2, 3 and 6
respectively, then the length of the line segment is : [JEE-MAIN Online 2013]
(1) 7 (2) 9 (3) 12 (4) 6

20. If two lines L1 and L2 in space, are defined by [JEE-MAIN Online 2013]


L1 = x   y    1 
z=  
 1 y + 
 and


L2 = x   y  1    

z = 1  y +  
 , then L1 is perpendicular to L2, for all non-negative reals  and , such that :

(1)  =  (2)  (3)   =1 (4)  +  = 0

21. The equation of a plane through the line of intersection of the planes x + 2y = 3, y – 2z+ 1 = 0,
and perpendicular to the first plane is : [JEE-MAIN Online 2013]
(1) 2x – y + 7z = 11 (2) 2x – y + 10 z = 11
(3) 2x – y – 9z = 10 (4) 2x – y – 10z = 9

22. 3, 2) and C (, 5, μ) in three


Let ABC be a triangle with vertices at points A (2, 3, 5), B (–1,
with the axes, then (, ) is
dimensional space. If the median through A is equally inclined
equal to : [JEE-MAIN Online 2013]
(1) (10, 7) (2) (7, 5) (3) (7, 10) (4) (5,7)
Corporate Office: NAIVEDHYAM, Plot No. SP-11, Old INOX, Indra Vihar, Kota (Raj.) 324005 47
Call: 0744-2799900 Online Partner UNACADEMY
23. The angle between the lines whose direction cosines satisfy the equations  + m + n = 0 and

2 = m2 + n2 is : [JEE-MAIN 2014]
   
(1) (2) (3) (4)
3 4 6 2

x 1 y  3 z  4
24. 
The image of the line  in the plane 2x – y + z + 3 = 0 is the line :
3 1 5
[JEE-MAIN 2014]
x 3 y5 z 2 x 3 y5 z  2
(1)   (2)  
3 1 5 3 1 5
x 3 y5 z 2 x 3 y5 z 2
(3)   (4)  
3 1 5 3 1 5

25. The equation of the plane containing the line 2x – 5y + z = 3 ; x + y + 4z = 5, and parallel to the
plane, x + 3y + 6z = 1, is : [JEE(Main)-2015]
(1) x + 3y + 6z = 7 (2) 2x + 6y + 12z = –13
(3) 2x + 6y + 12z = 13 (4) x + 3y + 6z = –7

26. The distance of the point (1, 0, 2) from the point of intersection of the line
x2 y 1 z  2
=  and the plane x – y + z = 16, is : [JEE(Main)-2015]
3 4 12
(1) 3 21 (2) 13 (3) 2 14 (4) 8

27. The distance of the point (1, –5, 9) from the plane x – y + z = 5 measured along the line
x = y = z is : [JEE(Main)-2016]
20 10
(1) (2) 3 10 (3) 10 3 (4)
3 3

28. If the image of the point P (1, –2, 3) in the plane, 2x + 3y – 4z + 22 = 0 measured parallel to
x y z
line,   is Q, then PQ is equal to : - [JEE (Main)-2017]
1 4 5
(1) 6 5 (2) 3 5 (3) 2 42 (4) 42

29. The distance of the point (1, 3, –7) from the plane passing through the point (1, –1, –1), having
x 1 y  2 z  4 x  2 y 1 z  7
normal perpendicular to both the lines   and   is :
1 2 3 2 1 1
[JEE (Main)-2017]
10 20 10 5
(1) (2) (3) (4)
74 74 83 83

Corporate Office: NAIVEDHYAM, Plot No. SP-11, Old INOX, Indra Vihar, Kota (Raj.) 324005 48
Call: 0744-2799900 Online Partner UNACADEMY
30. If L1 is the line of intersection of the planes 2x – 2y + 3z – 2 = 0, x – y + z + 1 = 0 and L2 is the
line of intersection of the planes x + 2y – z – 3 = 0, 3x – y + 2z – 1 = 0, then the distance of the
origin from the plane, containing the lines L1 and L2 is : [JEE (Main)-2018]
1 1 1 1
(1) (2) (3) (4)
2 4 2 3 2 2 2

31. The plane through the intersection of the planes x + y + z = 1 and 2x + 3y – z + 4 = 0 and
parallel to y-axis also passes through the point : [JEE (Main)-2019]
(1) (–3, 0, –1) (2) (3, 3, –1) (3) (3, 2, 1) (4) (–3, 1, 1)

32. The equation of the line passing through (–4, 3, 1), parallel to the plane x + 2y – z – 5 = 0 and
x 1 y  3 z  2
intersecting the line   is : [JEE (Main)-2019]
3 2 1
x  4 y  3 z 1 x4 y3 z 1
(1)   (2)  
1 1 1 3 1 1
x  4 y  3 z 1 x4 y3 z 1
(3)   (4)  
1 1 3 2 1 4

33. If the lines x = ay + b, z = cy + d and x = a'z + b', y = c'z +d' are perpendicular, then :
(1) cc' + a + a' = 0 (2) ab' + bc' + 1 = 0 [JEE (Main)-2019]
(3) aa' + c + c' = 0 (4) bb' + cc' + 1 = 0

x y z
34. The equation of the plane containing the straight the   and perpendicular to the plane
2 3 4
x y z x y z
containing the straight lines   and   is : [JEE (Main)-2019]
3 4 2 4 2 3
(1) 5x + 2y – 4z = 0 (2) x + 2y – 2z = 0
(3) 3x + 2y – 3z = 0 (4) x – 2y + z = 0

x  2 y  2 z 1
35. The plane passing through the point (4, –1, 2) and parallel to the lines   and
3 1 2
x 2 y 3 z 4
  also passes through the point: [JEE (Main)-2019]
1 2 3
(1) (1, 1, – 1) (2) (1, 1, 1) (3) (–1, –1, 1) (4) (–1, –1, –1)


36. Let A be a point on the line r  (1  3) ˆi  (  1) ˆj (2  5) kˆ and B(3, 2, 6) be a point in the

space. Then the value of  for which the vector AB is parallel to the plane x – 4y + 3z = 1 is:
[JEE (Main)-2019]
1 1 1 1
(1) (2) (3)  (4)
4 8 4 2

Corporate Office: NAIVEDHYAM, Plot No. SP-11, Old INOX, Indra Vihar, Kota (Raj.) 324005 49
Call: 0744-2799900 Online Partner UNACADEMY
37. The plane which bisects the line segment joining the points (–3, –3, 4) and (3, 7, 6) at right
angles, passes through which one of the following points ? [JEE (Main)-2019]
(1) (4, –1, 7) (2) (4, 1, –2) (3) (2, 1, 3) (4) (–2, 3, 5)

x 4 y 5 z 3
38. On which of the following lines lies the point of intersection of the line,  
2 2 1
and the plane, x + y + z = 2 ? [JEE (Main)-2019]
x  3 4  y z 1 x 2 y 3 z 3
(1)   (2)  
3 3 2 2 2 3
x 4 y 5 z 5 x 1 y  3 z  4
(3)   (4)  
1 1 1 1 2 5

39. The direction ratios of normal to the plane through the points (0, –1, 0) and (0,0,1) and making

an angle with the plane y – z + 5 = 0 are : [JEE (Main)-2019]
4
(1) 2 ,1,–1 (2) 2, 2,– 2 (3) 2 3 ,1 , –1 (4) 2, –1, 1

x  3 y  2 z 1
40. The plane containing the line  = and also containing its projection on the
2 1 3
plane 2x + 3y – z = 5, contains which one of the following points ? [JEE (Main)-2019]
(1) (2, 0, –2) (2) (2, 2, 0) (3) (–2, 2, 2) (4) (0, –2, 2)

x  3 y 1 z  6 x 5 y 2 z 3
41. Two lines   and   intersect at the point R. The reflection
1 3 1 7 6 4
of R in the xy-plane has coordinates: [JEE (Main)-2019]
(1) (–2, 4, 7) (2) (2, 4, 7) (3) (2, –4, –7) (4) (2, –4, 7)

42. If the point (2, , ) lies on the plane which passes through the points (3, 4, 2) and (7, 0, 6) and
is perpendicular to the plane 2x – 5y = 15, then 2 – 3 is equal to: [JEE (Main)-2019]
(1) 17 (2) 5 (3) 7 (4) 12

43. The perpendicular distance from the origin to the plane containing the two lines,
x 2 y2 z5 x 1 y  4 z  4
  and   is : [JEE (Main)-2019]
3 5 7 1 4 7
11
(1) 11 (2) 11 6 (3) (4) 6 11
6

Corporate Office: NAIVEDHYAM, Plot No. SP-11, Old INOX, Indra Vihar, Kota (Raj.) 324005 50
Call: 0744-2799900 Online Partner UNACADEMY
x 1 y  2 z  3 2 2
44. If an angle between the line,   and the plane, x – 2y – kz = 3 is cos 1   ,
2 1 2  3 
then a value of k is: [JEE (Main)-2019]

5 3 3 5
(1)  (2)  (3) (4)
3 5 5 3

45. Let S be the set of all real values of  such that a plane passing through the points (–2, 1, 1), (1, –2,
1) and (1, 1, – 2) also passes through the point (–1, –1, 1). The S is equal to:
[JEE (Main)-2019]
(1) {3, –3} (2) { 3,  3} (3) { 3} (4) [1, –1]

x 3 y – 2 z
46. The length of the perpendicular from the point (2, –1, 4) on the straight line  
10 –7 1
is [JEE (Main)-2019]
(1) less than 2 (2) greater than 4
(3) greater than 3 but less than 4 (4) greater than 2 but less than 3

47. The equation of a plane containing the line of intersection of the planes 2x – y – 4 = 0 and
y + 2z – 4 = 0 and passing through the point (1, 1, 0) is : [JEE (Main)-2019]
(1) x – 3y – 2z = –2 (2) 2x – z = 2 (3) x – y – z = 0 (4) x + 3y + z = 4

48. The vector equation of the plane through the line of intersection of the planes x + y + z = 1 and
2x + 3y + 4z = 5 which is perpendicular to the plane x – y + z = 0 is : [JEE (Main)-2019]
   
(1) r  (iˆ  k)
ˆ  2  0 (2) r  (iˆ  k)
ˆ  2  0 (3) r  (iˆ  k)
ˆ 2 0 (4) r  (iˆ  k)
ˆ 20

x 1 y  1 z  2
49. If the line,   meets the plane, x + 2y + 3z = 15 at a point P, then the distance
2 3 4
of P from the origin is : [JEE (Main)-2019]

7 9 5
(1) (2) (3) 2 5 (4)
2 2 2

Corporate Office: NAIVEDHYAM, Plot No. SP-11, Old INOX, Indra Vihar, Kota (Raj.) 324005 51
Call: 0744-2799900 Online Partner UNACADEMY

50. A plane passing through the points (0, –1, 0) and (0, 0, 1) and making an angle with the
4
plane y – z + 5 = 0, also passes through the point : [JEE (Main)-2019]

(1) ( 2,1, 4) (2) ( 2, 1, 4) (3) ( 2,1, 4) (4) ( 2, 1, 4)

x  2 y –1 z
51. The vertices B and C of a ABC lie on the line   such that BC = 5 units. Then
3 0 4
the area (in sq. units) of this triangle given that the point A(1, –1, 2) is : [JEE (Main)-2019]

(1) 34 (2) 2 34 (3) 5 17 (4) 6

52. Let P be the plane which contains the line of intersection of the planes, x + y +z – 6 =0 and
2x + 3y + z + 5 = 0 and it is perpendicular to the xy-plane. Then the distance of the point
(0, 0, 256) from P is equal to: [JEE (Main)-2019]

(1) 11/ 5 (2) 17 / 5 (3) 205 / 5 (4) 63 / 5

x y –1 z  1
53. If the length of the perpendicular from the point (, 0, ), (  0) to the line,   is
1 0 –1

3
, then  is equal to : [JEE (Main)-2019]
2
(1) 2 (2) – 2 (3) –1 (4) 1

54. Let A (3, 0, –1) , B(2, 10, 6) and C(1, 2, 1) be the vertices of a triangle and M be the midpoint
of AC. If G divides BM in the ratio 2 : 1, then cos (GOA) (O being the origin) is equal to :
[JEE (Main)-2019]
1 1 1 1
(1) (2) (3) (4)
15 6 10 30 2 15

55. If Q(0, –1, –3) is the image of the point P in the plane 3x – y + 4z = 2 and R is the point
(3, –1, –2), then the area (in sq. units) of PQR is : [JEE (Main)-2019]

91 65 91
(1) 2 13 (2) (3) (4)
4 2 2
Corporate Office: NAIVEDHYAM, Plot No. SP-11, Old INOX, Indra Vihar, Kota (Raj.) 324005 52
Call: 0744-2799900 Online Partner UNACADEMY
1 2
56. If the plane 2x – y + 2z + 3 = 0 has the distances and units from the planes 4x – 2y + 4z +  = 0
3 3
and 2x – y + 2z +  = 0, respectively, then the maximum value of  +  is equal to :
[JEE (Main)-2019]
(1) 9 (2) 15 (3) 5 (4) 13

x 1 y 1 z
57. A perpendicular is drawn from a point on the line   to the plane x + y + z = 3
2 1 1
such that the foot of the perpendicular Q also lies on the plane x – y + z = 3. Then the co-
ordinates of Q are : [JEE (Main)-2019]
(1) (2, 0, 1) (2) (–1, 0, 4) (3) (4, 0, – 1) (4) (1, 0, 2)

x – 2 y  1 z –1
58. If the line   intersects the plane 2x + 3y – z + 13 = 0 at a point P and the
3 2 –1
plane 3x + y + 4z = 16 at a point Q, then PQ is equal to [JEE (Main)-2019]

(1) 14 (2) 2 7 (3) 2 14 (4) 14

59. A plane which bisects the angle between the two given planes 2x – y + 2z – 4 = 0 and
x + 2y + 2z – 2 = 0, passes through the point : [JEE (Main)-2019]
(1) (2, 4, 1) (2) (1, –4, 1) (3) (2, –4, 1) (4) (1, 4, – 1)

60. The length of the perpendicular drawn from the point (2, 1, 4) to the plane containing the lines
 ˆ ˆ ˆ and r  (iˆ  ˆj)  (ˆi  ˆj  2k)
r  (i  j)  (iˆ  2ˆj  k) ˆ is : [JEE (Main)-2019]
1 1
(1) (2) 3 (3) (4) 3
3 3

61. Let P be a plane passing through the points (2, 1, 0), (4, 1, 1) and (5, 0, 1) and R be any point
(2, 1, 6). Then the image of R in the plane P is : [JEE (Main)-2020]
(1) (6, 5, 2) (2) (4, 3, 2) (3) (6, 5, –2) (4) (3, 4, –2)

x 3 y 8 z 3 x 3 y7 z6
62. The shortest distance between the lines   and   is :
3 1 1 3 2 4
7
(1) 3 (2) 2 30 (3) 30 (4) 3 30
2
[JEE (Main)-2020]

Corporate Office: NAIVEDHYAM, Plot No. SP-11, Old INOX, Indra Vihar, Kota (Raj.) 324005 53
Call: 0744-2799900 Online Partner UNACADEMY
 7 4 1 
63. The mirror image of the point (1, 2, 3) in a plane is  , ,  which of the following points
 3 3 3 
lies on this plane. [JEE (Main)-2020]
(1) (–1, –1, –1) (2) (–1,–1, 1) (3) (1, –1, 1) (4) (1, 1 ,1)

64. The projection of the line segment joining the points (1, –1, 3) and (2, –4, 11) on the line
joining the points (–1, 2, 3) and (3, –2, 10) is _______. [JEE (Main)-2020]

65. If the distance between the plane, 23x – 10y – 2x + 48 = 0 and the plane containing the lines
x 1 y  3 z 1 x  3 y  2 z 1 k
  and   (  R) is equal to , then k is equal to
2 4 3 2 6  663
____.
[JEE (Main)-2020]

66. The plane passing through the points (1, 2, 1), (2, 1, 2) and parallel to the line 2x = 3y, z = 1
also passes through the point : [JEE (Main)-2020]
(1) (0, 6, –2) (2) (–2, 0, 1) (3) (0, –6, 2) (4) (2, 0, –1)

67. A plane passing through the point (3,1,1) contains two lines whose direction ratios are 1, – 2, 2
and 2, 3, –1 respectively. If this plane also passes through the point (, – 3, 5), then  is equal
to : [JEE (Main)-2020]
(1) 5 (2) –5 (3) 10 (4) –10

68. The foot of the perpendicular drawn from the point (4, 2, 3) to the line joining the points
(1, –2, 3) and (1, 1, 0) lies on the plane: [JEE (Main)-2020]
(1) 2x + y – z = 1 (2) x – y – 2z = 1 (3) x – 2y + z = 1 (4) x + 2y – z = 1

69. The plane which bisects the line joining the points (4, –2, 3) and (2, 4, –1) at right angles also
passes through the point : [JEE (Main)-2020]
(1) (0, –1, 1) (2) (4, 0, –1) (3) (0, 1, –1) (4) (4, 0, 1)
 
70. Let a plane P contain two lines r  ˆi  (iˆ  ˆj), R and r  ˆj  ( ˆj  kˆ ), R .
If Q() is the foot of the perpendicular drawn from the point M(1, 0, 1) to P, then 3
() equal is________. [JEE (Main)-2020]

71. If the equation of a plane P, passing through the intersection of the planes, x + 4y – z + 7 = 0
and 3x + y + 5z = 8 is ax + by + 6z = 15 for some a, b  R, then the distance of the point
(3, 2, –1) from the plane P is ______. [JEE (Main)-2020]

72. The distance of the point (1, –2, 3) from the plane x – y + z = 5 measured parallel to the line
x y z
  is [JEE (Main)-2020]
2 3 6
1 7
(1) (2) 1 (3) 7 (4)
7 5

Corporate Office: NAIVEDHYAM, Plot No. SP-11, Old INOX, Indra Vihar, Kota (Raj.) 324005 54
Call: 0744-2799900 Online Partner UNACADEMY
x 1 y  3 z
73. If (a, b, c) is the image of the point (1, 2, –3) in the line,   , then a + b + c is
2 2 1
equal to : [JEE (Main)-2020]
(1) –1 (2) 3 (3) 2 (4) 1

74. If for some  R, the lines


x 1 y  2 z 1 x  2 y 1 z 1
L1 :   and L2 :   are coplanar, then the line L2 passes
2 1 1  5 1
through the point : [JEE (Main)-2020]
(1) (–2, 10, 2) (2) (2, –10, –2) (3) (10, 2, 2) (4) (10, –2, –2)

x 1 y 1 z
75. The shortest distance between the lines   and x + y + z + 1 = 0, 2x – y + z + 3 =
0 1 1
0 is: [JEE (Main)-2020]
1 1 1
(1) (2) 1 (3) (4)
2 2 3

76. A plane P meets the coordinate axes at A, B and C respectively. The centroid of ABC is given
to be (1, 1, 2). Then the equation of the line through this centroid and perpendicular to the plane
P is : [JEE (Main)-2020]
x 1 y 1 z  2 x 1 y 1 z  2
(1)   (2)  
1 2 2 2 2 1
x 1 y 1 z  2 x 1 y 1 z  2
(3)   (4)  
2 1 1 1 1 2

Corporate Office: NAIVEDHYAM, Plot No. SP-11, Old INOX, Indra Vihar, Kota (Raj.) 324005 55
Call: 0744-2799900 Online Partner UNACADEMY
EXERCISE # 4 (JA)
1. (a) A variable plane at a distance of 1 unit from the origin cuts the coordinate axis at A, B &
1 1 1
C. If the centroid of triangle ABC is D(x, y, z) satisfy the relation 2  2  2 = k, then
x y z
value of k is - [JEE 2005 (screening) 3]
(A) 3 (B) 1 (C) 1/3 (D) 9

(b) Find the equation of the plane containing the line 2x – y + z – 3 = 0, 3x + y + z = 5 and at a
1
distance of from the point (2, 1, –1) [JEE 05 (Mains) 2]
6

2. (a) A plane passes through (1, –2, 1) and is perpendicular to two planes 2x – 2y + z = 0 and
x – y + 2z = 4. The distance of the plane from the point (1, 2, 2) is
(A) 0 (B) 1 (C) 2 (D) 2 2

(b) Match the following

Column-I Column-II

1 1

 (y  1) dy   (1  y ) dy
2 2
(A) (P) 2
0 0

0 1

(B) A point (, , ) lies on the plane x + y + z = 2. (Q) 


1
1  xdx  
0
1  xdx

The value of  such that the vector


  
a = ˆi  ˆj  kˆ satisfies kˆ  (kˆ  a)  0

4
(C) In a triangle ABC, (R)
3
if cos A cos B + sin A sin B sin C = 1 then the
value of sin C is

(D) The set of values of a for which the lines (S) 1


x + y = |a|, ax – y = 4 intersect in the region
x > 0, y > 0, is the interval (a0, ). Then the value
of a0 is

Corporate Office: NAIVEDHYAM, Plot No. SP-11, Old INOX, Indra Vihar, Kota (Raj.) 324005 56
Call: 0744-2799900 Online Partner UNACADEMY
(c) Match the following
Column-I Column-II
5
(A) Let P be the plane passing through the point (2,1,–1) (P)
3
and perpendicular to the line of intersection of the
planes 2x + y – z = 3 and x + 2y + z = 2. Then the
distance from the point ( 3 , 2, 2) to the plane P is
n
 1 
(B) If Lim  tan 1  2  = t, then tan t is (Q) 1
n 
i 1  2i 
(C) The sides a,b,c of a triangle ABC are in A.P. If (R) 2/3
a b c  
cos1 = , cos2 = , cos3 = , then tan2 1 + tan2 3 =
bc ac ab 2 2
(D) Let L be the line passing through the point (0,1,0) and (S) 0
perpendicular to the plane x + 2y + 2z = 0. Then the
distance from the point (0,0,0) to the line L is [JEE 2006, 3+6+6]

3. (a) Consider the planes 3x – 6y – 2z = 15 and 2x + y – 2z = 5.


Statement-1 : The parametric equations of the line of intersection of the given planes are
x = 3 + 14t, y = 1 + 2t, z = 15t.
because
Statement-2 : The vector 14iˆ  2ˆj  15kˆ is parallel to the line of intersection of given
planes.
(A) Statement-1 is True, Statement-2 is True ; Statement-2 is a correct explanation for
Statement-1.
(B) Statement-1 is True, Statement-2 is True ; Statement-2 is NOT a correct explanation
for Statement-1.
(C) Statement-1 is True, Statement-2 is False.
(D) Statement-1 is False, Statement-2 is True.

(b) Consider the following linear equations ax + by + cz = 0; bx + cy + az = 0; cx + ay + bz = 0


Match the conditions / expressions in column I with statements in column II.
Column-I Column-II
(A) a + b + c  0 and (P) the equation represent planes meeting only
2 2 2
a + b + c = ab + bc + ca at a single point
(B) a + b + c = 0 and (Q) the equation represent the line x = y = z
a + b + c  ab + bc + ca
2 2 2

(C) a + b + c  0 and (R) the equation represent identical planes.


a + b + c  ab + bc + ca
2 2 2

(D) a + b + c = 0 and (S) the equation represent the whole of the


2 2 2
a + b + c = ab + bc + ca three dimensional space
[JEE 2007, 3+6]
Corporate Office: NAIVEDHYAM, Plot No. SP-11, Old INOX, Indra Vihar, Kota (Raj.) 324005 57
Call: 0744-2799900 Online Partner UNACADEMY
4. (a) Consider three planes
P1 : x – y + z = 1
P2 : x + y – z = –1
P3 : x – 3y + 3z = 2
Let L1, L2, L3 be the lines of intersection of the planes P2 and P3, P3 and P1, and P1 and P2,
respectively.
Statement-1 : At least two of the lines L1, L2 and L3 are non-parallel.
because
Statement-2 : The three planes do not have a common point.
(A) Statement-1 is True, Statement-2 is True ; Statement-2 is a correct explanation for
Statement-1.
(B) Statement-1 is True, Statement-2 is True ; Statement-2 is NOT a correct explanation
for Statement-1.
(C) Statement-1 is True, Statement-2 is False.
(D) Statement-1 is False, Statement-2 is True.

(b) Paragraph for Question (i) to (iii)


x 1 y  2 z 1 x  2 y  2 z 3
Consider the lines L1 = = = , L2 =  
3 1 2 1 2 3

(i) The unit vector perpendicular to both L1 and L2 is –


ˆi  7ˆj  7kˆ ˆi  7ˆj  5kˆ ˆi  7ˆj  5kˆ 7iˆ  7ˆj  kˆ
(A) (B) (C) (D)
99 5 3 5 3 99

(ii) The shortest distance between L1 and L2 is –


17 41 17
(A) 0 (B) (C) (D)
3 5 3 5 3

(iii) The distance of the point (1, 1, 1) from the plane passing through the point (–1, –2, –1) and
whose normal is perpendicular to both the lines L1 and L2 is –
2 7 13 23
(A) (B) (C) (D)
75 75 75 75
[JEE 2008, 3+4+4+4]

Corporate Office: NAIVEDHYAM, Plot No. SP-11, Old INOX, Indra Vihar, Kota (Raj.) 324005 58
Call: 0744-2799900 Online Partner UNACADEMY
 ˆ +  (3iˆ ˆj 5k)
ˆ .
5. (a) Let P(3, 2, 6) be a point in space and Q be a point on the line r  (iˆ ˆj 2 k)

Then the value of  for which the vector PQ is parallel to the plane x – 4y + 3z = 1 is –
1 1 1 1
(A) (B) – (C) (D) –
4 4 8 8

(b) A line with positive direction cosines passes through the point P (2, –1, 2) and makes equal
angles with the coordinate axes. The line meets the plane 2x + y + z = 9 at point Q. The
length of the line segment PQ equals –

(A) 1 (B) 2 (C) 3 (D) 2

(c) Let (x, y, z) be points with integer coordinates satisfying the system of homogeneous
equations :
3x – y – z = 0 ; –3x + z = 0 ; –3x + 2y + z = 0. Then the number of such points for which
x2 + y2 +z2  100 is [JEE 2009, 3+3+4]

x y z
6. (a) Equation of the plane containing the straight line   and perpendicular to the
2 3 4
x y z x y z
plane containing the straight lines   and   is
3 4 2 4 2 3
(A) x + 2y – 2z = 0 (B) 3x + 2y – 2z = 0
(C) x – 2y + z = 0 (D) 5x + 2y – 4z = 0

(b) If the distance of the point P(1,–2,1) from the plane x + 2y – 2z = , where > 0, is 5, then
the foot of the perpendicular from P to the plane is-
8 4 7  4 4 1  1 2 10  2 1 5
(A)  , ,   (B)  ,  ,  (C)  , ,  (D)  ,  , 
3 3 3  3 3 3 3 3 3  3 3 2

(c) If the distance between the plane Ax – 2y + z = d and the plane containing the lines
x 1 y  2 z  3 x 2 y3 z 4
  and   is 6 , then |d| is
2 3 4 3 4 5

Corporate Office: NAIVEDHYAM, Plot No. SP-11, Old INOX, Indra Vihar, Kota (Raj.) 324005 59
Call: 0744-2799900 Online Partner UNACADEMY
(d) Match the statements in Column-I with the values in Column-II.
Column-I Column-II
(A) A line from the origin meets the lines (p) –4
8
x
x  2 y 1 z  1
  and 3  y  3  z  1 at
1 2 1 2 1 1
P and Q respectively. If length PQ = d, then d2 is

(B) The values of x satisfying (q) 0


3
tan–1 (x + 3) – tan–1(x – 3) = sin–1   are
5
   
(C) Non-zero vectors a, b and c satisfy a.b = 0, (r) 4
       
(b a).(b c) = 0 and 2 | (b c) || (b a) | .
  
If a = b  4c , then the possible values of  are

(D) Let f be the function on [–,] given by (s) 5


 9x  x
f(0) = 9 and f(x) = sin   sin   for x  0. (t) 6
 2  2

2
The value of  f (x) dx is [JEE 2010, 3+5+3+(2+2+2+2)]
 
7. (a) The point P is the intersection of the straight line joining the points Q(2,3,5) and R(1,–1,4)
with the plane 5x – 4y – z = 1. If S is the foot of the perpendicular drawn from the point
T(2,1,4) to QR, then the length of the line segment PS is -
1
(A) (B) 2 (C) 2 (D) 2 2
2
(b) The equation of a plane passing through the line of intersection of the planes x + 2y + 3z = 2
2
and x – y + z = 3 and at a distance from the point (3, 1, –1) is
3
(A) 5x – 11y + z = 17 (B) 2 x + y = 3 2 – 1
(C) x + y + z = 3 (D) x – 2 y = 1 – 2
x 1 y 1 z x 1 y 1 z
(c) If the straight lines = = and = = are coplanar, then the
2 k 2 5 2 k
plane(s) containing these two lines is(are)
(A) y + 2z = –1 (B) y + z = –1 (C) y – z = –1 (D) y – 2z = –1
Corporate Office: NAIVEDHYAM, Plot No. SP-11, Old INOX, Indra Vihar, Kota (Raj.) 324005 60
Call: 0744-2799900 Online Partner UNACADEMY
x2 y 1 z
8. Perpendiculars are drawn from points on the line = = to the plane x + y + z = 3.
2 1 3
The feet of perpendiculars lie on the line [JEE-Advanced 2013, 2]
x y 1 z  2 x y 1 z  2
(A)   (B)  
5 8 13 2 3 5
x y 1 z  2 x y 1 z  2
(C)   (D)  
4 3 7 2 7 5

9. A line  passing through the origin is perpendicular to the lines

1 : (3 + t) î + (–1 + 2t) ˆj + (4 + 2t) k̂, – < t <

2 : (3 + 2s) î + (3 + 2s) ˆj + (2 + s) k̂, – < s <

Then, the coordinate(s) of the point (s) on 2 at a distance of 17 from the point of intersection
of  and 1is(are) – [JEE-Advanced 2013, 4, (–1)]
 7 5  7 8
(A)  , ,  (B) (–1, –1, 0) (C) (1, 1, 1) (D)  , , 
 3 3 3 9 9 9

y z y z
10. Two lines L1 : x = 5, = and L2 : x = , = are coplanar. Then  can take
3 2 1 2
value(s) [JEE-Advanced 2013, 3, (–1)]
(A) 1 (B) 2 (C) 3 (D) 4

x 1 y z3 x4 y3 z3


11. Consider the lines L1 = = = , L2 = = = and the planes
2 1 1 1 1 2
P1 : 7x + y + 2z = 3, P2 : 3x + 5y – 6z = 4. Let ax + by + cz = d be the equation of the plane
passing through the point of intersection of lines L1 and L2 and perpendicular to planes P1 and P2.
Match List-I with List-II and select the correct answer using the code given below the list.
List-I List-II
P. a= 1. 13
Q. b= 2. –3
R. c= 3. 1
S. d= 4. –2
Codes :
P Q R S
(A) 3 2 4 1
(B) 1 3 4 2
(C) 3 2 1 4
(D) 2 4 1 3 [JEE-Advanced 2013, 3, (–1)]

Corporate Office: NAIVEDHYAM, Plot No. SP-11, Old INOX, Indra Vihar, Kota (Raj.) 324005 61
Call: 0744-2799900 Online Partner UNACADEMY
12. From a point P(, , ), perpendiculars PQ and PR are drawn respectively on the lines
y = x, z = 1 and y = –x, z = –1. If P is such that QPR is a right angle, then the possible
value(s) of  is(are)
(A) 2 (B) 1 (C) –1 (D) – 2
[JEE(Advanced)-2014, 3]

13. In 3, consider the planes P1 : y = 0 and P2 : x + z = 1. Let P3 be a plane, different from P1 and
P2, which passes through the intersection of P1 and P2. If the distance of the point (0,1,0) from
P3 is 1 and the distance of a point (, , ) from P3 is 2, then which of the following relations is
(are) true ? [JEE 2015, 4M, –2M]
(A) 2 +  + 2 + 2 = 0 (B) 2 –  + 2 + 4 = 0
(C) 2 +  – 2 – 10 = 0 (D) 2 –  + 2 – 8 = 0

14. In 3, let L be a straight line passing through the origin. Suppose that all the points on L are at
a constant distance from the two planes P1 : x + 2y – z + 1 = 0 and P2 : 2x – y + z –1 = 0. Let M
be the locus of the feet of the perpendiculars drawn from the points on L to the plane P 1. Which
of the following points lie(s) on M ? [JEE 2015, 4M, –2M]
 5 2  1 1 1  5 1  1 2
(A)  0,  ,   (B)   ,  ,  (C)   , 0,  (D)   , 0, 
 6 3  6 3 6  6 6  3 3

15. Consider a pyramid OPQRS located in the first octant (x  0,y  0, z  0) with O as origin, and
OP and OR along the x-axis and the y-axis, respectively. The base OPQR of the pyramid is a
square with OP= 3. The point S is directly above the mid-point T of diagonal OQ such that TS = 3.
Then- [JEE(Advanced)-2016, 4(–2)]

(A) the acute angle between OQ and OS is
3
(B) the equation of the plane containing the triangle OQS is x – y = 0
3
(C) the length of the perpendicular from P to the plane containing the triangle OQS is
2
15
(D) the perpendicular distance from O to the straight line containing RS is
2

16. Let P be the image of the point (3, 1, 7) with respect to the plane x – y + z = 3. Then the
x y z
equation of the plane passing through P and containing the straight line   is
1 2 1
[JEE(Advanced)-2016, 3(–1)]
(A) x + y – 3z = 0 (B) 3x + z = 0 (C) x – 4y + 7z = 0 (D) 2x – y = 0

Corporate Office: NAIVEDHYAM, Plot No. SP-11, Old INOX, Indra Vihar, Kota (Raj.) 324005 62
Call: 0744-2799900 Online Partner UNACADEMY
17. The equation of the plane passing through the point (1, 1, 1) and perpendicular to the planes
2x + y – 2z = 5 and 3x – 6y – 2z = 7, is [JEE (Advanced)-2017]
(A) –14 x + 2y + 15 z = 3 (B) 14x + 2y + 15 z = 31
(C) 14 x + 2y – 15 z = 1 (D) 14 x – 2y + 15 z = 27

18. Let P1 : 2x + y – z = 3 and P2 : x + 2y + z = 2 be two planes. Then, which of the following


statement(s) is (are) TRUE ? [JEE (Advanced)-2018 Paper-1]
(A) The line of intersection of P1 and P2 has direction ratios 1, 2, –1
3x  4 1  3y z
(B) The line   is perpendicular to the line of intersection of P1 and P2
9 9 3
(C) The acute angle between P1 and P2 is 60º
(D) If P3 is the plane passing through the point (4, 2, –2) and perpendicular to the line of
2
intersection of P1 and P2, then the distance of the point (2, 1, 1) from the plane P3 is
3

19. Let P be a point in the first octant, whose image Q in the plane x + y = 3 (that is, the line
segment PQ is a perpendicular to the plane x + y = 3 and the mid-point of PQ lies in the plane
x + y = 3) lies on the z-axis. Let the distance of P from the x-axis be 5. If R is the image of P in
the xy-plane, then the length of PR is _______. [JEE (Advanced)-2018 Paper-2]

20. Three lines are given by



r  ˆi,   

r  (iˆ  ˆj),   and

r  v(iˆ  ˆj  k),
ˆ v
Let the lines cut the plane x + y + z = 1 at the points A, B and C respectively. If the area of the
triangle ABC is then the value of (6)2 equals__ [JEE(Advanced)-2019]

21. Let L1 and L2 be the following straight line.


x 1 y z 1 x 1 y z 1
L1 :   and L2 :  
1 1 3 3 1 1
Suppose the straight line
x   y 1 z  
L:  
l m 2
lies in the plane containing L1 and L2, and passes through the point of intersection of L1 and L2.
If the line L bisects the acute angle between the lines L1 and L2, then which of the following
statements is/are TRUE? [JEE(Advanced)-2020]
(A)  –  = 3 (B) l + m = 2 (C)  –  = 1 (D) l + m = 0

22. Let , , ,  be real number such that 2 + 2 + 2  0 and +  = 1. Suppose the point (3, 2, –
1) is the mirror image of the point (1, 0, – 1) with respect to the plane x + y + z = . Then
which of the following statements is/are TRUE ? [JEE(Advanced)-2020]
(A) +  = 2 (B)  –  = 3 (C)  +  = 4 (D)  +  +  = 
Corporate Office: NAIVEDHYAM, Plot No. SP-11, Old INOX, Indra Vihar, Kota (Raj.) 324005 63
Call: 0744-2799900 Online Partner UNACADEMY
EXERCISE # 5
MULTIPLE OBJECTIVE TYPE
1. The volume of a right triangular prism ABCA1B1C1 is equal to 3. If the position vectors of the
vertices of the base ABC are A(1, 0, 1) ; B(2,0, 0) and C(0, 1, 0) then the position vectors of the
vertex A1 can be:
(A) (2, 2, 2) (B) (0, 2, 0) (C) (0, – 2, 2) (D) (0, – 2, 0)

x  y z y z
2. Consider Lines L1 : = = , L2 : x = , = , plane P : 2x + 2y + z + 7 = 0.
1 2 2  2  
Let line L2 lies in plane P, then
(A)  = –7
(B)  = 7
(C) minimum distance between line L1 and plane P is 11.
23
(D) minimum distance between line L1 and plane P is
3

  
3. The point A (3,4,7), B (4,5,9) and C (1,2,–1) are three vertices of a parallelogram ABCD,
then-


(A) vector equation of AB is 4iˆ  5jˆ  9kˆ   ˆi  ˆj  2kˆ 
x4 x 5 z9
(B) cartesian equation of BC is = =
3 3 10
(C) coordinates of D are (0,1,–3)
(D) ABCD is rectangle

4. Let two planes P1 : 2x – y + z – 2 = 0 and P2 : x + 2y – z – 3 = 0 are given then-


(A) The equation of the plane through line of intersection of P1 = 0 and P2 = 0 and the point
(3,2,1) is x – 3y + 2z + 1 = 0
(B) The equation of the plane through line of intersection of P1 = 0 and P2 = 0 and the point
(3,2,1) is 3x – y + 2z – 9 = 0
(C) The equation of acute angle bisector plane of P1 = 0 and P2 = 0 is x – 3y + 2z + 1 = 0
(D) The equation of acute angle bisector plane of P1 = 0 and P2 = 0 is x + 3y + 2z + 2 = 0

5. A variable point P(4 cost, 4sint, 4sint) moves in space, now which of the following holds good?
(A) Point P moves on plane ax + by + cz + d = 0
(B) Point 'P' traces a circle.
(C) Area enclosed by P is 16 2 
(D) Point P cannot lie on a fixed plane.

Corporate Office: NAIVEDHYAM, Plot No. SP-11, Old INOX, Indra Vihar, Kota (Raj.) 324005 64
Call: 0744-2799900 Online Partner UNACADEMY
x y 1 z 1 x y 1 z 1
6. The projection of line  = on a plane 'P' is = = . If the plane P
2 2 1 1 1 1
passes through (k, –2, 0), then k is greater than -
(A) 2 (B) 3 (C) 5 (D) 4

7. A line segment has length 6 and direction ratios are –3,4,6, then the component of the line
vector are-
18 24 36 18 24 36
(A) , , (B) 27,–18,–54 (C) 27,–18,54 (D) , ,
61 61 61 61 61 61

8. Which of the following is (are) correct -


(A) If two lines in space are not intersecting, then they must be skew lines.
(B) If two lines are parallel to a plane 'P', then their direction ratios will be proportional
(C) If two lines are perpendicular to a plane 'P', then their direction ratios will be proportional
x 1 y 1 z
(D) Equation = = , where a, b, c are real parameters, represents a family of
a b c
concurrent lines in space

9. Given the equations of the line 3x – y + z + 1 = 0, 5x + y + 3z = 0. Then which of the following


is correct?
1 5
y z
x 8 8
(A) Symmetrical form of the equations of line is 
2 1 1
1 5
x y
(B) symmetrical form of the equations of line is 8 8 z
1 1 2
(C) equation of the plane through (2,1,4) and perpendicular to the given lines is 2x – y + z – 7 = 0
(D) equation of the plane through (2,1,4) and perpendicular to the given lines is x+y – 2z+5 = 0

10. Consider the family of planes x + y + z = c where c is a parameter intersecting the coordinate
axes at P,Q,R and ,, are the angles made by each member of this family with positive x,y
and z axis. Which of the following interpretations hold good for this family -
(A) each member of this family is equally inclined with the coordinate axes.
(B) sin2 + sin2 + sin2 = 1
(C) cos2 + cos2 + cos2 = 2
(D) for c = 3 area of the triangle PQR is 3 3 sq. units.

Corporate Office: NAIVEDHYAM, Plot No. SP-11, Old INOX, Indra Vihar, Kota (Raj.) 324005 65
Call: 0744-2799900 Online Partner UNACADEMY
11. Consider a plane P passing through A(,3,), B(–1,3,2) and C(7,5,10) and a straight line L with
positive direction cosines passing through A, bisecting BC and makes equal angles with the
coordinate axes. Let L1 be a line parallel to L and passing through origin. Which of the
following is(are) correct?
(A) The value of ( + ) is equal to 5.
x 1 y 1 z 1
(B) Equation of straight line L1 is   .
1 1 1
(C) Equation of the plane perpendicular to the plane P and containing line L1 is x – 2y + z = 0
(D) Area of triangle ABC is equal to 3 2 .

12. A line L passing through the point P(1,4,3), is perpendicular to both the lines
x 1 y3 z2 x2 y  4 z 1
= = and = = .
2 1 4 3 2 2
If the position vector of point Q on L is(a1,a2,a3) such that (PQ)2= 357, then (a1+a2+a3) can be-
(A) 16 (B) 15 (C) 2 (D) 1

MATRIX MATCH TYPE

13. P(0, 3, – 2); Q(3, 7, – 1) and R(1, – 3, – 1) are 3 given points. Let L1 be the line passing through

P and Q and L2 be the line through R and parallel to the vector V  ˆi  k.
ˆ
Column-I Column-II
(A) perpendicular distance of P from L2 (P) 7 3
(B) shortest distance between L1 and L2 (Q) 2
(C) area of the triangle PQR (R) 6
19
(D) distance from (0, 0, 0) to the plane PQR (S)
147

Corporate Office: NAIVEDHYAM, Plot No. SP-11, Old INOX, Indra Vihar, Kota (Raj.) 324005 66
Call: 0744-2799900 Online Partner UNACADEMY
EXERCISE # 6
x 3 y3 z
1. Find the equations of the two lines through the origin which intersect the line = =
2 1 1

at an angle of .
3

2. Let  : x + y – z – 4 = 0 be the equation of a plane and A be the point with position vector
ˆi  2ˆj  3kˆ . L is a line which passes through the point (1,2,3) with direction ratios 3,–1 and 4. If

the distance of the point A from the line L measured parallel to the plane  is d1 and the
distance of the point A from the plane  measured parallel to the line L is d2, then find the
value of d12  d 22 .

3. The three planes kx + y + z = 2, x + y – z = 3, x + 2z = 2 form a triangular prism and area of the


normal section (where normal section of the triangular prism is the plane parallel to the base of
the triangular prism) be k1. Then value of 2 14 (k.k1) is

x  6 y  10 z  14
4. The line   is the hypotenuse of an isosceles right angled triangle whose
5 3 8
opposite vertex is (7, 2, 4). Find the equation of the remaining sides.

5. (a) Consider a plane passing through three points A(a,0,0), B(0,b,0), C(0,0,c) with a > 0,
b > 0, c > 0. Let d be the distance between the origin O and the plane and m be the distance
between the origin O and the point M(a, b, c). If a, b, c vary in the range of any positive
2
m
numbers, then find the minimum value of   .
d
(b) Let A1,A2,A3,A4 be the areas of the triangular faces of a tetrahedron and h1,h2,h3,h4 be
corresponding altitudes of the tetrahedron. If volume of tetrahedron is 5 cubic units then
find the minimum value of (A1 + A2 + A3 + A4) (h1+ h2 + h3 + h4) (in cubic units).

6. If the angle between the planes given by 6x2 + 4y2 – 10z2 + 3yz + 4zx – 11xy = 0 is cos–1(k),
then the value of 'k' is equal to

7. Planes P1', P2',P3' are drawn parallel to the planes P1 : x + y + z = 3, P2 : x – y + z = 1 &


P3 : x + y – z = 2 respectively from the point (2,2,3). If d1, d2, d3 are distances of P1', P2', P3'
 1 1 
from (1,1,2) respectively then  d12  2  2  is equal to
 d 2 d3 

Corporate Office: NAIVEDHYAM, Plot No. SP-11, Old INOX, Indra Vihar, Kota (Raj.) 324005 67
Call: 0744-2799900 Online Partner UNACADEMY
8. Faces ABC and BCD of a tetrahedron ABCD meet at an angle of 30°. The area of face ABC is
120 and the area of face BCD is 80 and BC = 10, then the volume of tetrahedron is

9. Through a point P( , , ) a plane is drawn at right angle to OP to meet the axes in A, B, C. If


(OP) m
the area of ABC can be written as (where O is origin, m, n  N), then the value of
n...
(m2 + n2) is

10. (i) Points P1, P2, P3 ..... P10 are either lying along vertices or midpoints of the edges of a
tetrahedron as shown in the diagram, then the number of groups of four distinct points
(where each group of four points contains point P1) which lies on the same plane is equal
to

(ii) Let A, B, C, D be four non-coplanar points. Then the number of planes which are
equidistant from all the four points is equal to

Corporate Office: NAIVEDHYAM, Plot No. SP-11, Old INOX, Indra Vihar, Kota (Raj.) 324005 68
Call: 0744-2799900 Online Partner UNACADEMY
ANSWER KEY
EXERCISE # 1
1. D 2. A 3. D 4. D 5. A 6. B 7. D
8. B 9. A 10. A 11. C 12. B 13. C 14. C
15. C 16. D 17. A 18. C 19. B 20. B 21. C
22. A 23. B 24. A 25. C 26. C
27. (A) R,S; (B) Q; (C) Q,S; (D) P,S

EXERCISE # 2
x 1 y2 z3 1 1 1 1
1.  = 90º 2. 212 3. = = 4.  2 2 = 2
2 2 3 x 2
y z p
x  7 y  2 z 1
5. p = 3, (2, 1, –3); x + y + z = 0 6.  
22 5 4

7. (a) 3; (b) x + y – 2z + 1 = 0; (c) x – 2y + z = 5; (d) /3; (e) 4


2 x4 y 1 z7
8. 9. 11 10. = = 11. x – 2y + 2z – 1 = 0 ; 2 units
3 9 1 3
x4 y  14 z4
12. 9/2 13. 17 14. = = 15. (1, –2, –4)
3 10 4
x y z 19
16.   = 1, Area = sq. units 17. 2x + 3y + z + 4 = 0
2 3 5 2

EXERCISE # 3 (JM)
1. 4 2. 4 3. 4 4. 1 5. 3 6. 4 7. 1
8. 2 9. 3 10. 4 11. 2 12. 2 13. 4 14. 3
15. 3 16. 2 17. 3 18. 1 19. 1 20. 1,4 21. 2
22. 3 23. 1 24. 1 25. 1 26. 2 27. 3 28. 3
29. 3 30. 3 31. 3 32. 2 33. 3 34. 4 35. 2
36. 1 37. 2 38. 4 39. 1, 2 40. 1 41. 3 42. 3
43. 3 44. 4 45. 2 46. 3 47. 3 48. 2 49. 2
50. 1 51. 1 52. 1 53. 3 54. 1 55. 4 56. 4
57. 1 58. 3 59. 3 60. 4 61. 3 62. 4 63. 3
64. 8.00 65. 3.00 66. 2 67. 1 68. 1 69. 2 70. 5.00
71. 3.00 72. 2 73. 3 74. 2 75. 4 76. 2
Corporate Office: NAIVEDHYAM, Plot No. SP-11, Old INOX, Indra Vihar, Kota (Raj.) 324005 69
Call: 0744-2799900 Online Partner UNACADEMY
EXERCISE # 4 (JA)
1. (a) D; (b) 2x – y + z – 3 = 0 and 62x + 29y + 19z – 105 = 0
2. (a) D; (b) (A) Q,R; (B) P, (C) S, (D) P ; (c) (A) Q, (B) Q, (C) R, (D) P
3. (a) D; (b) (A) R; (B) Q, (C) P, (D) S
4. (a) D; (b) (i) B; (ii) D; (iii) C 5. (a) A; (b) C; (c) 7
6. (a) C; (b) A; (c) 6; (d) (A) t (B) p,r (C) q (D) r
7. (a) A; (b) A; (c) B,C 8. D 9. B,D 10. A,D
11. A 12. C 13. B,D 14. A,B 15. B,C,D 16. C 17. B
18. C,D 19. 8.00 20. 0.75 21. A,B 22. A,B,C

EXERCISE # 5
1. A,D 2. A,D 3. A,B,C 4. A,C 5. A,C 6. A,B,D 7. A,D

8. C,D 9. B,D 10. A,B,C 11. B,C,D 12. B,D

13. (A) R; (B) Q; (C) P ; (D) S

EXERCISE # 6
x y z x y z
1.   or   2. 10 3. 18
1 2 1 1 1 2

x 7 y2 z4 x 7 y2 z4


4.   ; = = 5. (a) 9 (b) 240
3 6 2 2 3 6

6. 0 7. 9 8. 320 9. 29 10. (i) 33 (ii) 7

Corporate Office: NAIVEDHYAM, Plot No. SP-11, Old INOX, Indra Vihar, Kota (Raj.) 324005 70
Call: 0744-2799900 Online Partner UNACADEMY
2019

100 Percentile 99.99 Percentile 99.98 Percentile 99.98 Percentile 99.97 Percentile 99.97 Percentile 99.96 Percentile 99.96 Percentile
HIMANSHU GAURAV SINGH GAURAV KRISHAN GUPTA SARTHAK ROUT VIBHAV AGGARWAL RITVIK GUPTA BHAVYA JAIN AYUSH PATTNAIK SAYANTAN DHAR
2019 (*SDCCP) 2020 (DLP) 2020 (CCP) 2019 (CCP) 2020 (DLP) 2020 (CCP) 2019 (CCP) 2020 (DLP)

You might also like